You are on page 1of 58

NSG6020- FINAL EXAM

REVIEW| Comprehensive
Questions and Answers 100%
Accuracy

Breast and Lymphatics


Statement by the patient would indicate a need for further teaching?

A. "The best time to check my breasts is a week before my cycle."


B. "I will check my breasts in the shower one week after my cycle."
C. "I will exam my breast in a clock-like sequence so that I don't miss any surface area."
D. "I will be sure to check my arm pits."

A
The week before a woman's cycle is not the best time to check for lumps because the breasts become
sore and tender and may also be "lumpy" a week prior to the cycle.
Answer 2 is incorrect because the week after a woman's cycle is the best time to do the examination, and
the shower allows for ease of palpation.
Answer 3 in incorrect because a clock-like sequence is the proper way to perform a BSE.
Answer 4 is incorrect because under the arms houses lymph nodes and the axillary tail of Spence, which
is a common site for breast cancer.
A pregnant woman who has breast implants asks the nurse if she can still breastfeed. What is the
nurse's best response?

A. "You should not have any problems breast feeding because your implants do not affect milk
production."
B. "When the breast implants are inserted they usually affect the milk glands, and breastfeeding is not
possible."
C. "This would depend on which type of implants were placed and which procedure was used by the
surgeon. Check with your surgeon to see if your milk production will be affected."
C
Milk production may vary depending on the procedure. Many women can still breastfeed after breast
augmentation. The surgeon would be able to determine this ability.
Answers 1 and 2 are incorrect because they may not be true. Only the surgeon can determine if milk
production will be affected.
Statement by the patient would indicate a need for further teaching?
A. "The best time to check my breasts is a week before my cycle."
B. "I will check my breasts in the shower one week after my cycle."
C. "I will exam my breast in a clock-like sequence so that I don't miss any surface area."
D. "I will be sure to check my arm pits."

A
The week before a woman's cycle is not the best time to check for lumps because the breasts become
sore and tender and may also be "lumpy" a week prior to the cycle.
Answer 2 is incorrect because the week after a woman's cycle is the best time to do the examination,
and the shower allows for ease of palpation.
Answer 3 in incorrect because a clock-like sequence is the proper way to perform a BSE.
Answer 4 is incorrect because under the arms houses lymph nodes and the axillary tail of Spence, which
is a common site for breast cancer.
A pregnant woman who has breast implants asks the nurse if she can still breastfeed. What is the
nurse's best response?

A. "You should not have any problems breast feeding because your implants do not affect milk
production."
B. "When the breast implants are inserted they usually affect the milk glands, and breastfeeding is not
possible."
C. "This would depend on which type of implants were placed and which procedure was used by the
surgeon. Check with your surgeon to see if your milk production will be affected."
C
Milk production may vary depending on the procedure. Many women can still breastfeed after breast
augmentation. The surgeon would be able to determine this ability.
Answers 1 and 2 are incorrect because they may not be true. Only the surgeon can determine if milk
production will be affected.
Which of the following statements is true regarding the internal structures of the breast? The breast
is:
A) mainly muscle, with very little fibrous tissue.
B) composed of fibrous, glandular, and adipose tissue.
C) composed mostly of milk ducts, known as lactiferous ducts.
D) composed of glandular tissue, which supports the breast by attaching to the chest wall.

ANS: B
The breast is composed of glandular tissue, fibrous tissue (including the suspensory ligaments), and
adipose tissue.
In performing a breast examination, the nurse knows that it is especially important to examine the
upper outer quadrant of the breast. The reason for this is that the upper outer quadrant is:
A) the largest quadrant of the breast.
B) the location of most breast tumors.
C) where most of the suspensory ligaments attach.
D) more prone to injury and calcifications than other locations in the breast.

ANS: B
The upper outer quadrant is the site of most breast tumors. In the upper outer quadrant, the nurse should
notice the axillary tail of Spence, the cone-shaped breast tissue that projects up into the axilla, close to
the pectoral group of axillary lymph nodes.
In performing an assessment of a woman's axillary lymph system, the nurse should assess which of
these nodes?
A) Central, axillary, lateral, and sternal nodes
B)Pectoral, lateral, anterior, and sternal nodes
C) Central, lateral, pectoral, and subscapular nodes
D)Lateral, pectoral, axillary, and suprascapular
nodes
ANS: C
The breast has extensive lymphatic drainage. Four groups of axillary nodes are present: (1) central, (2)
pectoral (anterior), (3) subscapular (posterior), and (4) lateral.
If a patient reports a recent breast infection, then the nurse should expect to find node
enlargement.
A)nonspecific
B)ipsilateral axillary
C)contralateral axillary
D) inguinal and cervical

ANS: B
The breast has extensive lymphatic drainage. Most of the lymph, more than 75%, drains into the
ipsilateral, or same side, axillary nodes.
A 9-year-old girl is in the clinic for a sports physical. After some initial shyness she finally asks,
"Am I normal? I don't seem to need a bra yet, but I have some friends who do. What if I never get
breasts?" The nurse's best response would be:
A) "Don't worry, you still have plenty of time to develop."
B) "I know just how you feel, I was a late bloomer myself. Just be patient and they will grow."
C) "You will probably get your periods before you notice any significant growth in your breasts."
D) "I understand that it is hard to feel different from your friends. Breasts usually develop between 8 and
10 years of age."
ANS: D
Adolescent breast development usually begins between 8 and 10 years of age. The nurse should not belittle
the girl's feelings by using statements like "don't worry" or by sharing personal experiences. The beginning
of breast development precedes menarche by about 2 years.
A patient contacts the office and tells the nurse that she is worried about her 10-year-old daughter
having breast cancer. She describes a unilateral enlargement of the right breast with associated
tenderness. She is worried because the left breast is not enlarged. What would be the nurse's best
response?
A) Tell the mother that breast development is usually fairly symmetric and she should be examined right
away.
B) Tell the mother that she should bring her daughter in right away because breast cancer is fairly
common in preadolescent girls.
C) Tell the mother that, although an examination of her daughter would rule out a problem, it is most
likely normal breast development.
D) Tell the mother that it is unusual for breasts that are first developing to feel tender because they
haven't developed much fibrous tissue.
ANS: C
Occasionally one breast may grow faster than the other, producing a temporary asymmetry. This may
cause some distress; reassurance is necessary. Tenderness is common also.
A 14-year-old girl is anxious about not having reached menarche. When taking the history, the nurse
should ascertain which of the following? The age:
A) she began to develop breasts
B) her mother developed breasts
C) she began to develop pubic hair
D) she began to develop axillary hair.

ANS: A
Full development from stage 2 to stage 5 takes an average of 3 years, although the range is 1.5 to 6 years.
Pubic hair develops during this time, and axillary hair appears 2 years after the onset of pubic hair. The
beginning of breast development precedes menarche by about 2 years. Menarche occurs in breast
development stage 3 or 4, usually just after the peak of the adolescent growth spurt, which occurs around
age 12 years. See Figure 17-6.
A woman is in the family planning clinic seeking birth control information. She states that her
breasts "change all month long" and that she is worried that this is unusual. What is the nurse's
best response?
A) Tell her that it is unusual. The breasts of nonpregnant females usually stay pretty much the same all
month long.
B) Tell her that it is very common for breasts to change in response to stress and that she should assess
her life for stressful events.
C) Tell her that, because of the changing hormones during the monthly menstrual cycle, cyclic breast
changes are common.
D) Tell her that breast changes normally occur only during pregnancy and that a pregnancy test is needed
at this time.
ANS: C
Breasts of the nonpregnant woman change with the ebb and flow of hormones during the monthly
menstrual cycle. During the 3 to 4 days before menstruation, the breasts feel full, tight, heavy, and
occasionally sore. The breast volume is smallest on days 4 to 7 of the menstrual cycle.
A woman has just learned that she is pregnant. What are some things the nurse should teach her
about her breasts?
A) She can expect her areolae to become larger and darker in color.
B) Breasts may begin secreting milk after the fourth month of pregnancy.
C) She should inspect her breasts for visible veins and report this immediately.
D) During pregnancy, breast changes are fairly uncommon; most of the changes occur after the birth.

ANS: A
The areolae become larger and grow a darker brown as pregnancy progresses, and the tubercles become
more prominent. (The brown color fades after lactation, but the areolae never return to the original color).
A venous pattern is prominent over the skin surface and does not need to be reported as it is an
expected finding. After the fourth month, colostrum, a thick, yellow fluid (precursor to milk) may be
expressed from the breasts.
The nurse is teaching a pregnant woman about breast milk. Which statement by the nurse is
correct?
A) "Your breast milk is present immediately after delivery of the baby."
B) "Breast milk is rich in protein and sugars (lactose) but has very little fat."
C) "The colostrum, which is present right after birth, does not contain the same nutrition as breast milk
does."
D) "You may notice a thick, yellow fluid expressed from your breasts as early as the fourth month
of pregnancy."
ANS: D
After the fourth month, colostrum may be expressed. This thick yellow fluid is the precursor of milk, and
it contains the same amount of protein and lactose but practically no fat. The breasts produce colostrum
for the first few days after delivery. It is rich with antibodies that protect the newborn against infection,
so breastfeeding is importat
A 65-year-old patient remarks that she just can't believe that her breasts sag so much. She states it
must be from lack of exercise. What explanation should the nurse offer her?
A) After menopause, only women with large breasts experience sagging.
B) After menopause, sagging is usually due to decreased muscle mass within the breast.
C) After menopause, a diet that is high in protein will help maintain muscle mass, which keeps the breasts
from sagging.
D) After menopause, the glandular and fat tissue atrophies, causing breast size and elasticity to diminish,
resulting in breasts that sag.
ANS: D
After menopause, the glandular tissue atrophies and is replaced with connective tissue. The fat envelope
atrophies also, beginning in the middle years and becoming marked in the eighth and ninth decades.
These changes decrease breast size and elasticity, so the breasts droop and sag, looking flattened and flabby
In examining a 70-year-old male patient, the nurse notices that he has bilateral gynecomastia.
Which of the following describes the nurse's best course of action?
A) Recommend that he make an appointment with his physician for a mammogram.
B) Ignore it; it is not unusual for men to have benign breast enlargement.
C) Explain that this condition may be the result of hormonal changes and recommend that he see his
physician.
D) Tell him that gynecomastia in men is usually associated with prostate enlargement and recommend
that he be screened thoroughly.
ANS: C
Gynecomastia may reappear in the aging male and may be due to testosterone deficiency.
During an examination of a 7-year-old girl, the nurse notices that the girl is showing breast budding.
What should the nurse do next?
A) Ask her if her periods have started.
B) Assess the girl's weight and body mass index (BMI).
C) Ask the girl's mother at what age she started to develop breasts.
D) Nothing; this is a normal finding.

ANS: B
Research has shown that girls with overweight or obese BMI levels have a higher occurrence of early
onset of breast budding (before age 8 years for African-American girls and age 10 years for white girls)
and early menarche.
The nurse is reviewing statistics regarding breast cancer. Which woman, aged 40 years in the United
States, has the highest risk for development of breast cancer?
A) African-
American B)White
C) Asian
D) American Indian

ANS: A
The incidence of breast cancer varies with different cultural groups. White women have a higher
incidence of breast cancer than African-American women starting at age 45 years; but African-American
women have a higher incidence before age 45 years. Asian, Hispanic, and American Indian women have
a lower risk for development of breast cancer (American Cancer Society, 2009-2010).
The nurse is preparing for a class in early detection of breast cancer. Which statement is true with
regard to breast cancer in African-American women in the United States?
A) Breast cancer is not a threat to African-American women.
B) African-American women have a lower incidence of regional or distant breast cancer than white
women.
C) African-American women are more likely to die of breast cancer at any age.
D) Breast cancer incidence in African-American women is higher than that of white women after age 45.

ANS: C
African-American women have a higher incidence of breast cancer before age 45 years than white
women, and are more likely to die of their disease. In addition, African-American women are
significantly more likely to be diagnosed with regional or distant breast cancer than are white women.
This racial difference in mortality rates may be related to insufficient use of screening measures and lack
of access to health care.
During a breast health interview, a patient states that she has noticed pain in her left breast. The
nurse's most appropriate response to this would be:
A) "Don't worry about the pain; breast cancer is not painful."
B) "I would like some more information about the pain in your left breast."
C) "Oh, I had pain like that after my son was born; it turned out to be a blocked milk duct."
D) "Breast pain is almost always the result of benign breast disease."

ANS: B
Breast pain occurs with trauma, inflammation, infection, or benign breast disease. The nurse will need to
gather more information about the patient's pain rather than make statements that ignore the patient's
concerns.
During a history interview, a female patient states that she has noticed a few drops of clear discharge
from her right nipple. What should the nurse do next?
A) Contact the physician immediately to report the discharge.
B) Ask her if she is possibly pregnant.
C) Ask her some additional questions about the medications she is taking.
D)Immediately obtain a sample for culture and sensitivity testing.
ANS: C
The use of some medications, such as oral contraceptives, phenothiazines, diuretics, digitalis,
steroids, methyldopa, and calcium channel blockers, may cause clear nipple discharge. Bloody or
blood-tinged discharge from the nipple, not clear, is significant, especially if a lump is also present. In
the pregnant female, colostrum would be a thick, yellowish liquid, and it would be expressed after the
fourth month of pregnancy.
During a physical examination, a 45-year-old woman states that she has had a crusty, itchy rash on
her breast for about 2 weeks. In trying to find the cause of the rash, which of these would be
important for the nurse to determine?
A) Is the rash raised and red?
B) Does it appear to be cyclic?
C) Where did it first appear—on the nipple, the areola, or the surrounding skin?
D) What was she doing when she first noticed the rash, and do her actions make it worse?

ANS: C
It is important for the nurse to determine where the rash first appeared. Paget's disease starts with a
small crust on the nipple apex and then spreads to the areola. Eczema or other dermatitis rarely starts
at nipple unless it results from breastfeeding. It usually starts on the areola or surrounding skin and then
spreads to the nipple. See Table 17-6.
A patient is newly diagnosed with benign breast disease. The nurse recognizes that which statement
about benign breast disease is true? The presence of benign breast disease:
A) makes it harder to examine the breasts.
B) frequently turns into cancer in a woman's later years.
C) is easily reduced with hormone replacement therapy.
D) is usually diagnosed before a woman reaches childbearing age.

ANS: A
The presence of benign breast disease (formerly fibrocystic breast disease) makes it harder to examine
the breasts; the general lumpiness of the breast conceals a new lump. The other statements are not true.
During an annual physical exam, a 43-year-old patient states that she doesn't perform monthly
breast self-examinations (BSE). She tells the nurse that she believes that mammograms "do a much
better job than I ever could to find a lump." The nurse should explain to her that:
A) BSEs may detect lumps that appear between mammograms.
B) breast self-examination is unnecessary until the age of 50 years.
C) she is correct, mammography is a good replacement for breast self-examination.
D) she doesn't need to perform breast self-examination as long as a physician checks her breasts yearly.

ANS: A
The monthly practice of breast self-examination, along with clinical breast examination and
mammograms are complementary screening measures. Mammography can reveal cancers too small to
be detected by the woman or by the most experienced examiner. However, interval lumps may become
palpable between mammograms.
During an interview, a patient reveals that she is pregnant. She states that she is not sure whether
she will breastfeed her baby and asks for some information about this. Which of these statements by
the nurse is accurate with regard to breastfeeding?
A) "Breastfed babies tend to be more colicky."
B) "Breastfeeding provides the perfect food and antibodies for your baby."
C)"Breastfed babies eat more often than infants on formula."
D)"Breastfeeding is second nature and every woman can do it."
ANS: B
Exclusively breastfeeding for 6 months provides the perfect food and antibodies for the baby, decreases
the risk of ear infections, promotes bonding, and provides relaxation
The nurse is reviewing risk factors for breast cancer. Which of these women have risk factors that
place them at a higher risk for breast cancer?
A) 37 year old who is slightly overweight
B) 42 year old who has had ovarian cancer
C) 45 year old who has never been pregnant
D) 65 year old whose mother had breast cancer

ANS: D
Risk factors for breast cancer include having a first-degree relative with breast cancer (mother, sister, or
daughter) and being older than 50 years. Refer to Table 17- 2 for other risk factors.
During an examination of a woman, the nurse notices that her left breast is slightly larger than her
right breast. Which of these statements is true about this finding?
A) Breasts should always be symmetric.
B) This finding is probably due to breastfeeding and is nothing to worry about.
C) This finding is not unusual, but the nurse should verify that this change is not new.
D) This finding is very unusual and means she may have an inflammation or growth.

ANS: C
The nurse should notice symmetry of size and shape. It is common to have a slight asymmetry in size;
often the left breast is slightly larger than the right. A sudden increase in the size of one breast signifies
inflammation or new growth.
The nurse is assisting with a self-breast examination clinic. Which of these women reflect abnormal
findings during the inspection phase of breast examination?
A) Woman whose nipples are in different planes (deviated)
B) Woman whose left breast is slightly larger than her right
C)Nonpregnant woman whose skin is marked with linear striae
D)Pregnant woman whose breasts have a fine blue network of veins visible under the skin
ANS: A
The nipples should be symmetrically placed on the same plane on the two breasts. With deviation in
pointing, an underlying cancer causes fibrosis in the mammary ducts, which pulls the nipple angle
toward it. The other examples are normal findings. See Table 17-3.
During the physical examination, the nurse notices that a female patient has an inverted left nipple.
Which statement regarding this is most accurate?
A) Normal nipple inversion is usually bilateral.
B) A unilateral inversion of a nipple is always a serious sign.
C) It should be determined whether the inversion is a recent change.
D) Nipple inversion is not significant unless accompanied by an underlying palpable mass.

ANS: C
The nurse should distinguish a recently retracted nipple from one that has been inverted for many years
or since puberty. Normal nipple inversion may be unilateral or bilateral and usually can be pulled out
(i.e., it is not fixed). Recent nipple retraction signifies acquired disease. See Table 17-3.
The nurse is performing a breast examination. Which of these statements best describes the
correct procedure to use when screening for nipple and skin retraction during a breast
examination? Have the woman:
A) bend over and touch her toes.
B) lie down on her left side and notice any retraction.
C) shift from a supine position to a standing position; notice any lag or retraction.
D) slowly lift her arms above her head and note any retraction or lag in movement.

ANS: D
Direct the woman to change position while checking the breasts for skin retraction signs. First ask her to
lift her arms slowly over her head. Both breasts should move up symmetrically. Retraction signs are due
to fibrosis in the breast tissue, usually caused by growing neoplasms. The nurse should notice if there is a
lag in movement of one breast.
The nurse is palpating a female patient's breasts during an examination. Which of these positions
is most likely to make significant lumps more distinct during breast palpation?
A) Supine with arms raised over her head
B) Sitting with arms relaxed at the sides
C) Supine with arms relaxed at the sides
D) Sitting with arms flexed and fingertips touching shoulders

ANS: A
The nurse should help the woman to a supine position, tuck a small pad under the side to be palpated,
and help the woman raise her arm over her head. These maneuvers will flatten the breast tissue and
displace it medially. Any significant lumps will then feel more distinct
Which of these clinical situations would the nurse consider to be outside normal limits?
A) A patient has had one pregnancy. She states that she believes she may be entering menopause. Her
breast examination reveals breasts that are sof t and sag slightly.
B) A patient has never been pregnant. Her breast examination reveals large pendulous breasts that have a
firm, transverse ridge along the lower quadrant in both breasts.
C) A patient has never been pregnant. She reports that she should begin her period tomorrow. Her breast
examination reveals breast tissue that is nodular and somewhat engorged. She states that the examination
was slightly painful.
D) A patient has had two pregnancies and she breastfed both of her children. Her youngest child is now
10 years old. Her breast examination reveals breast tissue that is somewhat sof t and she has a small
amount of thick yellow discharge from both nipples.
ANS: D
In nulliparous women, normal breast tissue feels firm, smooth, and elastic; after pregnancy, the tissue
feels softer and looser. If any discharge appears, the nurse should note its color and consistency. Except
in pregnancy and lactation, discharge is abnormal. Premenstrual engorgement is normal, and consists of
a slight enlargement, tenderness to palpation, and a generalized nodularity. A firm, transverse ridge of
compressed tissue in the lower quadrants, known as the inframammary ridge, is especially noticeable in
large breasts.
A patient states during the interview that she noticed a new lump in the shower a few days ago. It
was on her left breast near her axilla. The nurse should plan to:
A) palpate the lump first.
B) palpate the unaffected breast first.
C) avoid palpating the lump because it could be a cyst, which might rupture.
D) palpate the breast with the lump first but plan to palpate the axilla last.

ANS: B
If the woman mentions a breast lump she has discovered herself, the nurse should examine the
unaffected breast first to learn a baseline of normal consistency for this individual.
The nurse has palpated a lump in a female patient's right breast. The nurse documents this as a
small, round, firm, distinct, lump located at 2 o'clock, 2 cm from the nipple. It is nontender and
fixed. There is no associated retraction of skin or nipple, no erythema, and no axillary
lymphadenopathy. Which of these statements reveals the information that is missing from the
documentation? It is missing information about:
A) the shape of the lump.
B) the lump's consistency.
C) the size of the lump.
D) whether the lump is solitary or multiple.

ANS: C
If the nurse feels a lump or mass, he or she should note these characteristics: (1) location, (2) size—judge
in centimeters in three dimensions: width × length × thickness, (3) shape, (4) consistency, (5) motility, (6)
distinctness, (7) nipple, (8) the skin over the lump, (9) tenderness, and (10) lymphadenopathy.
The nurse is conducting a class about breast self-examination (BSE). Which of these statements
indicates proper BSE technique?
A) The best time to perform BSE is in the middle of the menstrual cycle.
B) The woman needs to do BSE only bimonthly unless she has fibrocystic breast tissue.
C) The best time to perform BSE is 4 to 7 days after the first day of the menstrual period.
D) If she suspects that she is pregnant, the woman should not perform a BSE until her baby is born.

ANS: C
The nurse should help each woman establish a regular schedule of self-care. The best time to conduct
breast self-examination is right after the menstrual period, or the fourth through seventh day of the
menstrual cycle, when the breasts are the smallest and least congested. Advise the pregnant or menopausal
woman who is not having menstrual periods to select a familiar date to examine her breasts each month,
for example, her birth date or the day the rent is due.
The nurse is preparing to teach a woman about breast self-examination (BSE). Which statement by
the nurse is correct?
A) "BSE is more important than ever for you because you have never had any children."
B) "BSE is so important because one out of nine women will develop breast cancer in her lifetime."
C) "BSE on a monthly basis will help you feel familiar with your own breasts and their normal variations."
D) "BSE will save your life because you are likely to find a cancerous lump between mammograms."

ANS: C
The nurse should stress that a regular monthly self-examination will familiarize her with her own breasts
and their normal variations. This is a positive step that will reassure her of her healthy state. While
teaching, the nurse should focus on the positive aspects of breast self-examination and should avoid citing
frightening mortality statistics about breast cancer. This may generate excessive fear and denial that
actually obstructs a woman's self-care action.
A 55-year-old postmenopausal woman is being seen in the clinic for a yearly examination. She is
concerned about changes in her breasts that she has noticed over the past 5 years. She states that her
breasts have decreased in size and that the elasticity has changed so that her breasts seem "flat and
flabby." The nurse's best reply would be:
A) "This change occurs most often because of long-term use of bras that do not provide enough support
to the breast tissues."
B) "This is a normal change that occurs as women get older. It is due to the increased levels of
progesterone during the aging process."
C) "Decreases in hormones after menopause causes atrophy of the glandular tissue in the breast. This is a
normal process of aging."
D) "Postural changes in the spine make it appear that your breasts have changed in shape. Exercises to
strengthen the muscles of the upper back and chest wall will help to prevent the changes in elasticity
and size."
ANS: C
The hormonal changes of menopause cause the breast glandular tissue to atrophy, making the breasts
more pendulous, flattened, and sagging.
A 43-year-old woman is at the clinic for a routine examination. She reports that she has had a breast
lump in her right breast for years. Recently, it has begun to change in consistency and is becoming
harder. She reports that 5 years ago her physician evaluated the lump and determined that it "was
nothing to worry about." The examination validates the presence of a mass in the right upper outer
quadrant at 1 o'clock, approximately 5 cm from the nipple. It is firm, mobile, nontender, with
borders that are not well defined. The nurse's recommendation to her is:
A) "Because of the change in consistency of the lump, it should be further evaluated by a physician."
B) "The changes could be related to your menstrual cycles. Keep track of changes in the mass each
month."
C) "This is probably nothing to worry about because it has been present for years and was determined to
be noncancerous at that time."
D) "Because you are experiencing no pain and the size has not changed, continue to monitor the lump
and return to the clinic in 3 months."
ANS: A
A lump that has been present for years and is not exhibiting changes may not be serious but still should
be explored. Any recent change or new lump should be evaluated. The other responses are not correct.
During a discussion about breast self-examination with a 30-year-old woman, which of these
statements by the nurse is most appropriate?
A) "The best time to examine your breasts is during ovulation."
B) "Examine your breasts every month on the same day of the month."
C) "Examine your breasts shortly after your menstrual period each month."
D) "The best time to examine your breasts is immediately before menstruation."

ANS: C
The best time to conduct breast self-examination is shortly after the menstrual period when the breasts
are the smallest and least congested.
The nurse is discussing breast self-examination with a postmenopausal woman. The best time for
postmenopausal women to perform breast self-examination is:
A) the same day every month.
B)daily, during the shower or
bath.
C) 1 week after her menstrual period.
D) every year with her annual gynecologic examination.

ANS: A
Postmenopausal women are no longer experiencing regular menstrual cycles but need to continue to
perform breast self-examination on a monthly basis. Choosing the same day of the month is a helpful
reminder to perform breast self-examination.
While inspecting a patient's breasts, the nurse finds that the left breast is slightly larger than the
right with the presence of Montgomery's glands bilaterally. The nurse should:
A) palpate over the Montgomery's glands, checking for drainage.
B) consider these normal findings and proceed with the examination.
C) ask extensive history questions regarding the woman's breast asymmetry.
D)
continue with examination and then refer the patient for further evaluation of the Montgomery's glands.
ANS: B
Normal findings of the breast include one breast (most often the left) slightly larger than the other and
the presence of Montgomery's glands across the areola.
During an examination, the nurse notes a supernumerary nipple just under the patient's left
breast. The patient tells the nurse that she always thought it was a mole. Which statement about
this finding is correct?
A) It is a normal variation and not a significant finding.
B) It is a significant finding and needs further investigation.
C) It also contains glandular tissue and may leak milk during pregnancy and lactation.
D) The patient is correct—it is actually a mole that happens to be located under the breast.

ANS: A
A supernumerary nipple looks like a mole, but close examination reveals a tiny nipple and areola. It is not
a significant finding.
While examining a 75-year-old woman, the nurse notices that the skin over her right breast is
thickened and the hair follicles are exaggerated. This condition is known as:
A) dimpling.
B)retraction.
C) peau d'orange.
D) benign breast disease.

ANS: C
This condition is known as peau d'orange. Lymphatic obstruction produces edema, which thickens the
skin and exaggerates the hair follicles. The skin has a pig-skin or orange-peel look, and this condition
suggests cancer.
When a breastfeeding mother is diagnosed with a breast abscess, which of these instructions from
the nurse is correct? The mother needs to:
A) continue to nurse on both sides to encourage milk flow.
B)discontinue nursing immediately to allow for healing.
C)temporarily discontinue nursing on affected breast and manually express milk and discard it.
D)temporarily discontinue nursing on affected breast but can manually express milk and give it to the
baby.
ANS: C
With a breast abscess, the patient must temporarily discontinue nursing on the affected breast, manually
express the milk, and discard it. Nursing can continue on the unaffected side.
A new mother calls the clinic to report that part of her left breast is red, swollen, tender, very hot,
and hard. She has a fever of 101° F. She has also had symptoms of the flu, such as chills, sweating,
and feeling tired. The nurse notices that she has been breastfeeding for 1 month. From her
description, what condition does the nurse suspect?
A) Mastitis
B) Paget's disease
C) Plugged milk duct
D) Mammary duct ectasia
ANS: A
The symptoms describe mastitis, which stems from infection or stasis caused by a plugged duct. A
plugged duct does not have infection present. (See Table 17-7.) Refer to Table 17-6 for descriptions of
Paget's disease and mammary duct ectasia.
During a breast examination on a female patient, the nurse notices that the nipple is flat, broad, and
fixed. The patient states it "started doing that a few months ago." This finding suggests:
A) dimpling.
B) a retracted nipple.
C) nipple inversion.
D)deviation in nipple
pointing.
ANS: B
The retracted nipple looks flatter and broader, like an underlying crater. A recent retraction suggests
cancer, which causes fibrosis of the whole duct system and pulls in the nipple. It also may occur with
benign lesions such as ectasia of the ducts. The nurse should not confuse retraction with the normal
long-standing type of nipple inversion, which has no broadening and is not fixed.
A 54-year-old man comes to the clinic with a "horrible problem." He tells the nurse that he has just
discovered a lump on his breast and is fearful of cancer. The nurse knows that which statement
about breast cancer in males is true?
A) Breast masses in men are difficult to detect because of minimal breast tissue.
B) Breast cancer in men rarely spreads to the lymph nodes.
C) One percent of all breast cancer occurs in men.
D) Most breast masses in men are diagnosed as gynecomastia.

ANS: C
One percent of all breast cancer occurs in men. Early spread to axillary lymph nodes occurs due to
minimal breast tissue.
The nurse is assessing the breasts of a 68-year-old woman and discovers a mass in the upper outer
quadrant of the left breast. When assessing this mass, the nurse keeps in mind that characteristics of
a cancerous mass include which of the following? Select all that apply.
A) Nontender mass
B) Dull, heavy pain on palpation
C) Rubbery texture and mobile
D)Hard, dense, and immobile
E) Regular border
F) Irregular, poorly delineated border

ANS: A, D, F
Cancerous breast masses are solitary, unilateral, nontender, masses. They are solid, hard, dense, and
fixed to underlying tissues or skin as cancer becomes invasive. Their borders are irregular and poorly
delineated. They are often painless, although the person may have pain. They are most common in upper
outer quadrant. A dull, heavy pain on palpation and a mass with a rubbery texture and a regular border
are characteristics of benign breast disease.
The nurse is examining a 62-year-old man and notes that he has gynecomastia bilaterally. The nurse
should explore his history for which related conditions? Select all that apply.
A) Obesity
B)Malnutrition
C)Hyperthyroidis
m
D) Type 2 diabetes mellitus
E) Liver disease
F) History of alcohol abuse

ANS: A, C, E, F
Gynecomastia occurs with obesity, Cushing's syndrome, liver cirrhosis, adrenal disease, hyperthyroidism,
and numerous drugs: alcohol and marijuana use, estrogen treatment for prostate cancer, antibiotics
(metronidazole, isoniazid), digoxin, ACE inhibitors, diazepam, and tricyclic antidepressants.

Which of the following statements is true regarding the internal structures of


the breast? The breast is made up of:
Fibrous, glandular, and adipose tissues.

Clinical breast exam should be performed every


- 3 years
-
GU WOMEN
MENORRHAGIA
- INREASED AMOUNT OF BLEEDING
METORRHAGIA
- BLEEDNG BETWEEN PERIODS
CERVICAL CANCER SCREENING WITH PAP SHOULD BE DONE
EVERY
- 3 YEARS
Normal ovary size
- 2-3CM
What is the confirmatory test for HIV?
A. ELISA
B. Western Blot
C. HIV polymerease chain reaction test
D. HIV antibody
B. Remember that the Western Blot is the confirmatory test for HIV but ELISA is the initial
test. If this test is positive then you confirm with the Western Blot.
What medication would a patient with HIV be taking everyday for the prevention
of pneumocystis carinii pneumonia (pneumocystis jirovecci)?
A. Bactrim
B. Penicillin
C. Doxycyline
D. Erythromcycin

- A. Bactrim must be taken daily for the prevention of pneumocytis carinii


pneumonia.
A patient comes into your office complaining of night sweats, weight loss, and fever.
You want to test for:
A. HIV
B. Syphilis
C. pneumonia
D. chancroid
Remember early HIV looks like the flu.
How long does it take a patient with HIV to serconvert?
A. 6 months to 1 year
B. 3 months to 1 year
C. 3 weeks to 6 months
D. Over one year
C. 3 weeks to 6 months - that is why everyone that has had a needle stick will have an
initial test then the person will come back in 6 months to retest.
What is a normal T-cell (CD4) count?
A. 800
B. 400
C. 200
D. None

- A. 800

In a patient with HIV if the viral load is high and the CD4 is low you know:
A. That they are stable
B. That they are severely immunocomprimised
C. There immune response is high.

- B. These patients have no immunity to fight infection and the virus is running rampant.

Chancroid is a:
A. Gram positive organism
B. Gram negative bacillus
C. Co-factor for HIV transmission
D. Both B and C

- B. Gram negative bacillus (hemophylus ducreyi). High rate of HIV identified among
individuals with chancroid. Estimated 10% of patients are also infected with syphilis
and HIV.
The degree to which those who have a disease screen/test positive is known as:
A. Specificity
B. Sensitivity

- B. Sensitivity: Think how many people are positive. Don't get tricked. This has nothing
to do with how many people might be negative.
If a patient complains of pain upon urination and does not have a urinary tract
infection you suspect:
A. Chlamydia
B. Syphilus
C. PID
D. Trichimoniasis

- A. Chlamydia: Remember the two sexually transmitted diseases that cause pain upon
urination (dysuria) are chlamydia and gonorrhea.
Patients who are diagnosed with gonorrhea are also treated for following because
of high rates of coinfection:
A. Mycoplasma pneumoniae
B. Chlamydia trachomatis
C. Syphilis
D. Pelvic inflammatory disease

- B. Chlamydia ( question 16 in Leik)

Human papilloma virus (HPV) infection of the larynx has been associated with:
A. Laryngeal cancer
B. Esophageal stricture
C. Cervical cancer
D. Metaplasia of squamous cells

- A. Laryngeal cancer (page 23)

The pap smear result on a 20 year old sexually active student who uses
condoms inconsistently show a large amount of inflammation. Which of the
following is the best follow-up.
A. The NP needs to do cervical cultures to verify the presence of gonorrhea
B. Treat the patient with metronidazole vaginal cream over the phone
C. Call the patient and tell her she needs a repeat pap smear in 6 months.
D. Advise her to use a betadine douche at H (half strength) for three days.

- C. A- Gonorrhea can cause cervicitis in women

Fitz Hugh Curtis Syndrome is associated with which of the following infections:
A. Syphilis
B. Chlamydia trachomatis
C. Herpes gentitalis
D. Lymhogranuloma venereum

- B. Chlamydia trachomatis: Fitz Hugh Curtis syndrome consists of persistent right upper
quadrant abdominal pain, perihepatitis and genital tract infection. Neisseria
gonorrheoeae and chlamydia trachomatis have been identified as the causative agents.
A 13 year old adolescent who is not sexually active is brought in by her mother for
an immunization update and physical examination. According to the mother her
daughter had two doses of hepatitis 1 year ago. All of the following are indicated
for this visit except:
A. Hapatitis B Vaccine
B. Tetanus vaccine
C. Screen for depression
D. HIV test

- D. HIV test. The question does not tell you that the patient has any risk factors
for HIV.
The ELISA and Western Blot tests are both used to test for the HIV virus. Which of
the following statements is correct.
A. They are both tests to detect viral RNA
B. A positive ELISA screening does not mean the person has HIV infection
C. They are both tests used to detect for antibodies against HIV virus.
D. They are both the best diagnostic tests for HIV

- B. A positive ELISA screening does not mean the person has HIV infection. The
ELISA test is always followed by Western blot test to confirm diagnosis.
All of the following are infections affecting the vagina or labia except:
A. Bacterial Vaginosis
B. Candidiasis
C. Trichomoniasis
D. Chlamydia Trachomatis

- D. Chlamydia Trachomatis: Initially infects the cervix and urethra. Signs of


infection include vaginal discharge or a burning sensation with urination.
The KOH prep (potassium hydroxide) will help you diagnose all of the following
conditions except:
A. Tinea infections
B. Candida albicans infections of the skin
C. Bacterial vaginosis
D. Atypical bacterial infections

- D. Atypical bacterial infections: A KOH test is done to determine whether a fungus


is causing the skin infection. Samples from an infected area are
treated with KOH which disolves skin cells a nd leaves behind fungus cells that can
be seen on a microscope. It is used to diagnose thrush, tinea vesicolor, ringworm, and
bacterial vaginosis.
KOH potassium hydroxide skin scraping will reveal

- hyphae

Trichomoniasis characteristics of vaginal discharge

- foul, frothy, grayish, strawberry spots, motile porotozoa

Bacterial vaginosis characteristics

- fishy, white-yellow, creamy, clue cells and whiff test on wet prep

Chlamydia characteristics/gonorrhea

- yellowish, no odor, purulent discharge, numerous white cells on wet prep

All of the following are reportable diseases except:


A. Lyme disease
B. Gonorrhea
C. Nongonococcal urethritis
D. Syphilis

- C. Nongonococcal urethritis

A 20 year old woman who is sexually active complains of copious milk like vaginal
discharge. On microscopy the slide reveals a large number of mature squamous
epithelial cells. The vaginal pH is 5.o. There are very few leukocytes and no red
blood cells seen on the wet smear. Which of the following is most likely.
A. Atrophic vaginits
B. Bacterial Vaginosis
C. Trichomoniasis
D. This is a normal finding

- B. Diagnosis of BV includes 3 of 4 Amsel criteria (1) white, thick adherent discharge,


(2) pH more than 4.5, (3) positive whiff test, (4) clue cells greater than 20% on wet
mount (epithelial cells dotted with large numbers of bacteria that obscure cell borders),
plus (5) Gram stain.
All of the following are clinical findings associated with syphilis except:
A. Condyloma lata
B. Condyloma acuminata
C. Painless chancre
D. Rashes on the palms of the hands and the soles of the feet
- B. Condyloma acuminata refers to an epidermal manifestation attributed to
epidermotropic huan papilloma virus (HPV)
The differential diagnosis for genital ulcerations includes all of the following
except:
A. Syphilis
B. Genital herpes
C. Chancroid
D. Molluscum contagiosum

- D. Molluscum contagiosum is a viral skin infection. appears on the face, neck


armpits, arms, and hands. Other common places include the genitals, abdomen, and
inner thigh. They often begin a small firm dome shaped growths, the surface feels
smooth, waxy, or pearly, are flesh colored or pink, have a dimple in the center, may
be filled with a waxy or cheesy substance, and are painless. scratching or pricking
can spread the virus,.
Which type of hepatitis infection is more likely to result in cirrhosis of the liver and
the risk of developing heaptocellular carcinoma.
A. Hepatitis A virus
B. Hepatitis B Virus
C. Hepatitis C virus
D. Both B and C

- D. Of the primary hepatitis viruses only B and C are associated with


hepatocellular cancer.
A 19 year old male has recently been diagnosed with acute hepatitis B. He is
sexually active and is monogamous and reports using condoms inconsistently.
Which of the following is recommended for his male sexual partner, who was also
recently tested for hepatitis with the following result HBsAG (-); anti-HBs (-);
anti-HVC (-); anti HAV (+).
A. Hepatitis B Vaccination
B. Hepatitis B immuno globulin
C. Hepatitis B vaccination and hepatitis immune globulin
D. No vaccination is needed at this time.

- C. Hepatitis B surface antigen (HBsAg) is a marker for infectivity. If positive it indicates


either an acute or chronic hepatitis B infection. Antibody to hepatitis surface antigen
(anti-HBs) is a marker of immunity. Antibody to hapatitis B core antigen (anti-HBc) is a
marker of acute, chronic, or resolved HBV infection. It may be used in prevaccination
testing to determine previous exposure to HBV. Interpretation of the hepatitis B panel
for test results: A negative HBsAg and a negative anti-HBc and a negative anti-HBs
indicates the patient is susceptible - not immune has not been infected and is still at risk
for future infection - need vaccination. Interpretation of the hepatitis C anti HCV
screening test that is negative indicates that the patient is not infected.
Which of the following is recommended for the treatment of a patient with bacterial
vaginosis?
A. Azithromycin (Zithromax)
B. Doxycyline (Dynapin)
C. Ceftriaxone (Rocephin)
D. Metronidazole (Flagyl)

- D. Treatment for BV is flagyl either orally or vaginally or treatment with


clindamycin (cleocin) either orally or vaginally. Treatment for the partner is not
recommended by the CDC, since there is no decrease recurrences with partner
treatment and no effect on cure rates.
All of the following sexually transmitted diseases can become disseminated
if not treated except:
A. Neissieria gonorrhoeae
B. Treponema pallidum
C. HIV
D. Chlamydia Trahomatis

- D. The key to answering this question is to understand the meaning behind


disseminated - think wide spread or systemic. Although chlamydia seems like an
obvious candidate since it can ascend to the uterus and affect a variety of sites
including the eyes and the rectum, the specific type of chlamydia that causes genital
infections is not generally thought to cause systemic infection. The predominant
infections are urethritis, cervicitis, and proctititis, but chlamydia infection can spread
locally to the bartholin glands, endosapinges, or epididymis. Up to 40% of untreated
chlamydial cervicitis cases will ascend into the upper genital tract where
considerable tubal damage can occur with very few symptoms.
A homeless middle aged male complains of migratory arthritis and dysuria.
Currently, his right knee is swollen and painful. On examination, green colored
purulent discharge is noted. Which of the following do you most likely suspect?
A. Gonorrhea
B. Chlamydia
C. Nongonococcal urethritis
D. Acute epididymitis

- A. Symptoms of gonorrhea usually appear 3-5 days after the infection however in men
symptoms may take up to one month to appear. Early symptoms include dysuria,
increased urination, frequency, urgency, white/yellow/or green penile drainage, red or
edematous penile urethra, and tender and swollen testes. If the infection spreads
systemically fever, rash, and arthritis like symptoms may occur.
Which of the following methods is used to diagnose gonorrheal proctitis?
A. Serum chlamydia titer
B. Gen-Probe
C. Thayer Martin culture
D. PRP (rapid plasma reagent) and VDRL (venereal disease research lab test)

- C. Thayer martin selective agar is an enriched medium for the selective isolation of
Neisseria species obtained by cervical culture. A Gram stain of the discharge smear can
also be used and shows gram negative diplococci and WBC
A 34 year old female is diagnosed with pelvic inflammatory disease. The cervical
gen probe result is positive for Neisseria gonorrhoeae and negative for chlamydia
trachomatis. All of the following treatments are true regarding the management
of this patient except:
A. This patient should be treated for chlamydia even though the gen-probe for chlamydia
is negative
B. Ceftriaxone 250 mg IM and doxycyline 100 mg po BID x 14 days are appropriate
treatment for this patient?
C. Advise the patient to return to the clinic for repeat pelvic examination in 48 hours
D. Repeat Gen-Probe test for chlamydia to ensure that the previous test was not a false
negative result

- D. Repeating the Gen-probe test for chlamydia is not recommended. The 2010 CDC
STD treatment guidelines for pelvic inflammatory disease state: PID comprises a
spectrum of inflammatory disorders of the upper female genital tract, including any
combination of endometritis, salpingitis, tubo- ovarian abscess, and pelvic peritonitis.
Delay in diagnosis and treatment probably contributes to inflammatory sequelae in the
upper reproductive tract. Treatment should be initiated as soon as the presumptive
diagnosis has been made, because prevention of long term sequelae is dependent on
early administration of appropriate antibiotics. All regimens used to treat PID sould also
be effective against N. gonorrhoeae and C. trachomatis because negative endocerivcal
screening for these organisms does not rule out upper reproductive tract infection. Out
patient oral therapy can be considered for women with mild to moderately severe acute
PID because the clinical outcomes among women treated with oral therapy are similar o
those treated with perenteral therapy. RECOMMENDED REGIMEN:
Ceftriaxone 250 mg I.M. in a single dose coupled with doxycylcine 100 mg orally
twice a day for 14 days with or without metronidazole 500 mg orally twice daily for
14 days.
A 25 year old woman comes into your office complaining of dysuria, pruritis,
and purulent vaginal dischage. Pelvic examination reveals a cervix with punctate
superficial petechiae (strawberry cervix) and irritated reddened vulvar area,
with frothy discharge. Microscopic examination of the discharge reveals
unicellular organisms. The correct pharmacological treatment for this condition
is:
A. Oral metronidazole (Flagyl)
B. Ceftriaxone sodium (Rocephin) injection
C. Doxycycline hyclate (vibramycin)
D. Clotrimazole (Gyne-Lotrimin) cream or suppositories

- A. A single dose of metronidazole is effective in the majority of cases of trichomonas


infections. The strawberry cervix is considered to be selectively associated with
Trichomonas. The etiology is the parastitic protozoan flagellate, T. vaginalis.
Symptoms include: fouls smelling vaginal discharge (often fishy), burning and
soreness of the vulva, perineum, and thighs, dyspareunia and dysuria. Wet prep
microscopic examination should reveal highly motile cells, slightly larger than
leukocytes, smaller than epithelial cells.
Prophylaxis for pneumocystis carinii pneumonia includes all of the following
drugs except:
A. Trimethoprim-sulfamethoxazole (Bactrim)
B. Dapsone
C. Aerosolized pentamidine
D. Nedocromil sodium Tilade

- D. Nedocromil sodium is a mast cell stabilizer used for the treatment of moderate
inflammation of the bronchial tubes and other allergy symptoms. Pneumocystis
pneumonia is the most common opportunitic infection in people with HIV. Drugs
used to treat PCP include bactrim (first line to treat pnumocystis pneumonia), dapson
(antibiotic traditionally used to treat leprosy), pentamidine (antimicrobial used to
prevent pneumoystis pneumonia), and atovaquone (naphthoquinones used to treat
pneumoystis pneumonia, toxoplasmosis, malaria, and babesia).
Which of the following is recommended for treatment by the CDC for a case of
uncomplicated gonorrheal and chlamydia infection?
A. Metronidazole 250 mg po three times daily for seven days
B. Valacylcovir 500 mg po bid for 10 days
C. Azithromycin 1 g orally or Doxyclycline 100 mg twice daily for seven days
D. One dose of oral Fluconazole 150 mg

- C. Azithromycin 1 g orally or doxycycline 100 mg orally twice daily for 7 days is


based on the CDC 2010 guidelines for treatment.
A patient diagnosed with bacterial vaginosis should be advised that her sexual
partner be treated with:
A. Ceftriazone (Rocephin) 250 mg with doxycycline 100 mg po bid x 14 days
B. Flagyl 500 mg BID x 7 days with 1 dose of azithromycin
C. Her partner does not need to be treated
D. Lotrimin cream to the penis twice daily for 14 days

- C. The usual medical regimen for treatment is the antibiotic flagyl 400 mg twice daily
every 12 hours for seven days. Treatment of sexual partners is not warranted.
A 38 year old active Asian woman with a history of infertility treatment and
severe endometriosis complains to the nurse practitioner in the emergency
department of right sided pelvic pain that is steadily getting worse. She also
reports small amounts of yellow vaginal discharge on her underwear for the last
week. The patient reports expecting her period which has not appeared in the last
two months. Which of the following should be performed initially?
A. Follicle stimulating hormone and culture and sensitivity testing for vaginal
discharge
B. Serum quantitative pregnancy test and quantitative cervical Gen-Probe testing
C. Pelvic ultrasound and serum quantitative pregnancy test
D. Microscopy of vaginal discharge and complete blood count with white cell
differential?

- C. The most critical step in beginning the workup is to have a high clinical suspicion
for ectopic pregnancy (in any woman of childbearing years). Bedside pelvic
sonography testing is the imaging test of choice to investigate early pregnancy. Patients
with indeterminite ultrasound findings should have a beta-hCG level drawn and should
be followed up closely with gynecology to monitor serial beta hcg levels and
ultrasonography.
Which of the following is the most important differential diagnosis to consider in
a 38 year old sexually active Asian woman with a history of infertility treatment
and severe endometrosis with right sided pelvic pain that is steadily getting worse
and small amounts of yellow colored discharge?
A. Tubal ectopic pregnancy and pelvic inflammatory disease
B. Mucopurulent cervicitis and tubal ectopic pregnancy
C. Human papilloma virus infection of the cervix and pelvic inflammatory disease
D. Ovarian cysts and severe endometriosis

- A. Her positive physical findings of left adnexal tenderness and discharge suggest
an ectopic pregnancy versus pelvic inflammatory disease. Amenorrhea should be
treated as a pregnancy until proven otherwise.
Vaginal candidiasis is best diagnosed in the primary care arena by the following
method:
A. Microscopy
B. Tzanck smear
C. KOH smear whiff test
D. Clinical findings only

- C. Generally wet preps use 1 to 2 slides to evaluate candida. under a microscope


observe for presence and number of wbc's, trichomonads, candidal hphea, or clue cells.
Yeast and hyphae may also be present on a wet prep. KOH prep is made by adding a
drop of saline suspension of vaginal discharge to a drop of KOH solution. The KOH
lyses epithelial cells in 5 to 15 minutes and allows easier visualization of candidal
hyphae.
*Tzanck test is scraping of an ulcer base to look for tazanck cells, sometimes also called
the chickenpox skin test and the herpes skin test.
For which diagnosis is the consideration of whether the patient has a bubo
necessary.
A. Chancroid
B. Syphilis
C. Genital herpes
D. Gonorrhea

- A. Remember a Bubo is a swollen lymph node in the genital area often seen with
chancroid.
A 21 year old Latino male comes into your office presenting with a sore on his penis
x2 weeks. On exam, you note an indurated chancre that is painless, along with mild
regional lymphadenopathy. Based on your diagnosis, which of the following is your
most appropriate order?
A. Ceftriaxone
B. PCN G
C. Azithromycin
D. Acylcolovir

- B. PCN

the nurse practitioner knows that asymptomatic shedding should be considered


with which STD/STI?
A. Syphilis
B. Chlamydia
C. HSV-2
D. Gonorrhea

- C. HSV-2

In teaching a patient about seroconversion with HIV the nurse practitioner


knows:
A. Most patients seroconvert within 3-4 weeks of exposure
B. Once seroconversion occurs the diagnosis of AIDS is usually made and
medication is started in most patients
C. Enough antigens have built up in the blood by 6 months to usually detect a virus
D. Once seroconversion occurs, the patient may or may not show symptoms.

- D

All of these are considered part of HIV/AIDS except:


A. Fatigue/vague abdominal pain
B. Weight loss
C. Night Sweats
D. Fever
- A. Fatigue/vague abdominal pain

The pap smear result on a 20 year old sexually active student who uses condoms
inconsistently shows a large amount of inflammation. Which of the following is
the best follow-up?
A. The NP needs to do cervical cultures to verify the presence of gonorrhea
B. Treat the patient with metronidazole vaginal cream
C. Call the patient and tell her she needs a repeat pap smear in six months
D. Advise her to use a betadine douche at half strength for three days.

- A - cultures should be taken at the time of the Pap smear as the patient may not return
for later diagnostic testing.

MALE GU
A normal prostate is described as
- Rubbery and non tender
A 32 y.o. male comes into your office complaining of a superficial, painful ulcer,
surrounded by an erythematous halo on the shaft of his penis with a unilateral
bubo. You suspect:
A. Chancroid
B. Lymphogranuloma venereum
C. condylomata acuminata
D. molluscum contagiosum
- A. Chancroid: Painful lesion. Only other sexually transmitted lesion that will be painful
is herpes.

NEUROLOGICAL SYSTEM
The two parts of the nervous system are the:
Central and peripheral.
The wife of a 65-year-old man tells the nurse that she is concerned because she has
noticed a change in her husband's personality and ability to understand. He also cries
very easily and becomes angry. The nurse recalls that the cerebral lobe responsible for
these behaviors is the lobe.
Frontal
Which statement concerning the areas of the brain is true?
The hypothalamus controls body temperature and regulates sleep.
The area of the nervous system that is responsible for mediating reflexes is the:
Spinal cord.
While gathering equipment after an injection, a nurse accidentally received a prick from
an improperly capped needle. To interpret this sensation, which of these areas must be
intact?
Lateral spinothalamic tract, thalamus, and sensory cortex
A patient with a lack of oxygen to his heart will have pain in his chest and possibly in the
shoulder, arms, or jaw. The nurse knows that the best explanation why this occurs is
which one of these statements?
The sensory cortex does not have the ability to localize pain in the heart; consequently,
the pain is felt elsewhere.
The ability that humans have to perform very skilled movements such as writing is
controlled by the:
Corticospinal tract.
A 30-year-old woman tells the nurse that she has been very unsteady and has had
difficulty in maintaining her balance. Which area of the brain that is related to these
findings would concern the nurse?
Cerebellum
Which of these statements about the peripheral nervous system is correct? The
peripheral nerves carry input to the central nervous system by afferent fibers and
away from the central nervous system by efferent fibers.
A patient has a severed spinal nerve as a result of trauma. Which statement is true in
this situation?
The adjacent spinal nerves will continue to carry sensations for the dermatome served
by the severed nerve.
A 21-year-old patient has a head injury resulting from trauma and is unconscious.
There are no other injuries. During the assessment what would the nurse expect to find
when testing the patient's deep tendon reflexes?
Reflexes will be normal.
A mother of a 1-month-old infant asks the nurse why it takes so long for infants to learn
to roll over. The nurse knows that the reason for this is: Myelin is needed to conduct
the impulses, and the neurons of a newborn are not yet myelinated.
During an assessment of an 80-year-old patient, the nurse notices the following: an
inability to identify vibrations at her ankle and to identify the position of her big toe,
a slower and more deliberate gait, and a slightly impaired tactile sensation. All other
neurologic findings are normal. The nurse should interpret that these findings
indicate:
Normal changes attributable to aging.
A 70-year-old woman tells the nurse that every time she gets up in the morning or
after she's been sitting, she gets "really dizzy" and feels like she is going to fall over.
The nurse's best response would be:
"You need to get up slowly when you've been lying down or sitting."
During the taking of the health history, a patient tells the nurse that "it feels like the
room is spinning around me." The nurse would document this finding as:
Vertigo.
When taking the health history on a patient with a seizure disorder, the nurse assesses
whether the patient has an aura. Which of these would be the best question for obtaining
this information?
"Do you have any warning sign before your seizure starts?"
While obtaining a health history of a 3-month-old infant from the mother, the nurse asks
about the infant's ability to suck and grasp the mother's finger.
What is the nurse assessing? Reflexes
In obtaining a health history on a 74-year-old patient, the nurse notes that he drinks
alcohol daily and that he has noticed a tremor in his hands that affects his ability to hold
things. With this information, what response should the nurse make?
"Does the tremor change when you drink alcohol?"
A 50-year-old woman is in the clinic for weakness in her left arm and leg that she has
noticed for the past week. The nurse should perform which type of neurologic
examination?
Complete neurologic examination
During an assessment of the CNs, the nurse finds the following: asymmetry when the
patient smiles or frowns, uneven lifting of the eyebrows, sagging of the lower eyelids,
and escape of air when the nurse presses against the right puffed cheek. This would
indicate dysfunction of which of these CNs?
Motor component of CN VII
The nurse is testing the function of CN XI. Which statement best describes the
response the nurse should expect if this nerve is intact? The patient: Moves the head
and shoulders against resistance with equal strength.
During the neurologic assessment of a "healthy" 35-year-old patient, the nurse asks
him to relax his muscles completely. The nurse then moves each extremity through
full range of motion. Which of these results would the nurse expect to find?
Mild, even resistance to movement
When the nurse asks a 68-year-old patient to stand with his feet together and arms at
his side with his eyes closed, he starts to sway and moves his feet farther apart. The
nurse would document this finding as:
Positive Romberg sign.
The nurse is performing an assessment on a 29-year-old woman who visits the clinic
complaining of "always dropping things and falling down." While testing rapid
alternating movements, the nurse notices that the woman is unable to pat both of her
knees. Her response is extremely slow and she frequently misses. What should the
nurse suspect?
Dysfunction of the cerebellum
During the taking of the health history of a 78-year-old man, his wife states that he
occasionally has problems with short-term memory loss and confusion: "He can't
even remember how to button his shirt." When assessing his sensory system, which
action by the nurse is most appropriate? Before testing, the nurse would assess the
patient's mental status and ability to follow directions.
The assessment of a 60-year-old patient has taken longer than anticipated. In testing
his pain perception, the nurse decides to complete the test as quickly as possible. When
the nurse applies the sharp point of the pin on his arm several times, he is only able to
identify these as one "very sharp prick." What would be the most accurate explanation
for this?
This response is most likely the result of the summation effect.
The nurse is performing a neurologic assessment on a 41-year-old woman with a
history of diabetes. When testing her ability to feel the vibrations of a
tuning fork, the nurse notices that the patient is unable to feel vibrations on the great
toe or ankle bilaterally, but she is able to feel vibrations on both patellae. Given this
information, what would the nurse suspect?
Peripheral neuropathy
The nurse places a key in the hand of a patient and he identifies it as a penny. What
term would the nurse use to describe this finding?
Astereognosis (inability to identify objects correctly, and it occurs in sensory cortex
lesions)
The nurse is testing the deep tendon reflexes of a 30-year-old woman who is in the
clinic for an annual physical examination. When striking the Achilles heel and
quadriceps muscle, the nurse is unable to elicit a reflex. The nurse's next response
should be to:
Ask the patient to lock her fingers and pull.
In assessing a 70-year-old patient who has had a recent cerebrovascular accident,
the nurse notices right-sided weakness. What might the nurse expect to find when
testing his reflexes on the right side?
Hyperactive reflexes
When the nurse is testing the triceps reflex, what is the expected response? Extension of
the forearm
The nurse is testing superficial reflexes on an adult patient. When stroking up the lateral
side of the sole and across the ball of the foot, the nurse notices the plantar flexion of
the toes. How should the nurse document this finding? Plantar reflex present
In the assessment of a 1-month-old infant, the nurse notices a lack of response to noise
or stimulation. The mother reports that in the last week he has been sleeping all of the
time, and when he is awake all he does is cry.
The nurse hears that the infant's cries are very high pitched and shrill. What should be
the nurse's appropriate response to these findings?
Refer the infant for further testing.
Which of these tests would the nurse use to check the motor coordination of an 11-
month-old infant?
Denver II
To assess the head control of a 4-month-old infant, the nurse lifts up the infant in a
prone position while supporting his chest. The nurse looks for what normal response?
The infant:
Raises the head, and arches the back.
While assessing a 7-month-old infant, the nurse makes a loud noise and notices the
following response: abduction and flexion of the arms and legs; fanning of the fingers,
and curling of the index finger and thumb in a C position, followed by the infant
bringing in the arms and legs to the body.
What does the nurse know about this response?
This reflex should have disappeared between 1 and 4 months of age.
To test for gross motor skill and coordination of a 6-year-old child, which of these
techniques would be appropriate? Ask the child to:
Hop on one foot.
During the assessment of an 80-year-old patient, the nurse notices that his hands
show tremors when he reaches for something and his head is always nodding. No
associated rigidity is observed with movement. Which of these statements is most
accurate?
These findings are normal, resulting from aging.
While the nurse is taking the history of a 68-year-old patient who sustained a head
injury 3 days earlier, he tells the nurse that he is on a cruise ship and is 30 years old.
The nurse knows that this finding is indicative of a(n): Decreased level of
consciousness.
The nurse is caring for a patient who has just had neurosurgery. To assess for
increased intracranial pressure, what would the nurse include in the assessment?
Level of consciousness, motor function, pupillary response, and vital signs During an
assessment of a 22-year-old woman who sustained a head injury from an automobile
accident 4 hours earlier, the nurse notices the following changes: pupils were equal, but
now the right pupil is fully dilated and nonreactive, and the left pupil is 4 mm and
reacts to light. What do these findings suggest?
Increased intracranial pressure
A 32-year-old woman tells the nurse that she has noticed "very sudden, jerky
movements" mainly in her hands and arms. She says, "They seem to come and go,
primarily when I am trying to do something. I haven't noticed them when I'm sleeping."
This description suggests:
Chorea.
During an assessment of a 62-year-old man, the nurse notices the patient has a
stooped posture, shuffling walk with short steps, flat facial expression, and pill-rolling
finger movements. These findings would be consistent with: Parkinsonism.
During an assessment of a 32-year-old patient with a recent head injury, the nurse
notices that the patient responds to pain by extending, adducting, and internally rotating
his arms. His palms pronate, and his lower extremities extend with plantar flexion.
Which statement concerning these findings is most accurate? This patient's response:
Is a very ominous sign and may indicate brainstem injury.
A 78-year-old man has a history of a cerebrovascular accident. The nurse notes that
when he walks, his left arm is immobile against the body with flexion of the shoulder,
elbow, wrist, and fingers and adduction of the shoulder. His left leg is stiff and
extended and circumducts with each step. What type of gait disturbance is this
individual experiencing?
Spastic hemiparesis
In a person with an upper motor neuron lesion such as a cerebrovascular accident,
which of these physical assessment findings should the nurse expect?
Hyperreflexia
A 59-year-old patient has a herniated intervertebral disk. Which of the following
findings should the nurse expect to see on physical assessment of this individual?
Hyporeflexia
A patient is unable to perform rapid alternating movements such as rapidly patting her
knees. The nurse should document this inability as:
Presence of dysdiadochokinesia.
The nurse knows that determining whether a person is oriented to his or her surroundings
will test the functioning of which structure(s)?
Cerebrum
During an examination, the nurse notices severe nystagmus in both eyes of a patient.
Which conclusion by the nurse is correct? Severe nystagmus in both eyes:
May indicate disease of the cerebellum or brainstem.
The nurse knows that testing kinesthesia is a test of a person's: Position
sense.
The nurse is reviewing a patient's medical record and notes that he is in a coma. Using
the Glasgow Coma Scale, which number indicates that the patient is in a coma?
6
A man who was found wandering in a park at 2 AM has been brought to the
emergency department for an examination; he said he fell and hit his head. During the
examination, the nurse asks him to use his index finger to touch the nurse's finger,
then his own nose, then the nurse's finger again (which has been moved to a different
location). The patient is clumsy, unable to follow the instructions, and overshoots the
mark, missing the finger. The nurse should suspect which of the following?
Acute alcohol intoxication
The nurse is assessing the neurologic status of a patient who has a late- stage brain
tumor. With the reflex hammer, the nurse draws a light stroke up the lateral side of the
sole of the foot and inward, across the ball of the foot. In response, the patient's toes fan
out, and the big toe shows dorsiflexion.
The nurse interprets this result as:
Positive Babinski sign, which is abnormal for adults.
A 69-year-old patient has been admitted to an adult psychiatric unit because his wife
thinks he is getting more and more confused. He laughs when he is found to be
forgetful, saying "I'm just getting old!" After the nurse completes a thorough
neurologic assessment, which findings would be indicative of Alzheimer disease?
Select all that apply.
b.Difficulty performing familiar tasks, such as placing a telephone call
cMisplacing items, such as putting dish soap in the refrigerator eRapid mood
swings, from calm to tears, for no apparent reason f.Getting lost in one's
own neighborhood
bone marrow- the musculoskeletal system functions to encase and protect the inner
vital organs, to support the body, to produce red blood cells in the bone marrow -
hematopoiesis) and to store minerals.
Fibrous bands running directly from one bone to another that strengthen the joint and
help prevent movement in undesirable directions are called: ligaments
The nurse notices that a woman in an exercise class is unable to jump rope. The nurse
is aware that to jump rope, one's shoulder has to be capable of: Circumduction is
defined as moving the arm in a circle around the shoulder. The other options are not
correct.
Of the 33 vertebrae in the spinal column, there are:
7 cervical 12 thorasic, 5 lumbar/sacral 3-4 cocccygeal vertebrae
an imaginary line connecting the highest point on each iliac crest would cross the
.............................vertebra
forth sacral
The nurse is explaining to a patient that there are shock absorbers in his back to
cushion the spine and to help it move. The nurse is referring to his: Intervertebral
disks are elastic fibrocartilaginous plates that cushion the spine similar to shock
absorbers and help it move. other landmarks are the The vertebral column is the spinal
column itself. The nucleus pulposus is located in the center of each disk. The vertebral
foramen is the channel, or opening, for the spinal cord in the vertebrae.
an injury to the rotator cuff involves which shoulder joint
glenohumeral joint which enclosed by a group of four powerful muscles and tendons
that support and stabilize it- other definitions are: the nucleus pulposus is located in the
center of each intervertebral disk. The articular processes are projections in each
vertebral disk that lock onto the next vertebra, thereby stabilizing the spinal column.
The medial epicondyle is located at the elbow.
A patient is visiting the clinic for an evaluation of a swollen, painful knuckle. The
nurse notices that the knuckle above his ring on the left hand is swollen and that he is
unable to remove his wedding ring. This joint is called the
joint.
metacarpophalangeal joint.
2. The interphalangeal joint is located distal to the metacarpophalangeal joint.
3. The tarsometatarsal and tibiotalar joints are found in the foot and ankle. The nurse
is assessing a patient's ischial tuberosity. To palpate the ischial tuberosity, the nurse
knows that it is best to have the patient:
The ischial tuberosity lies under the gluteus maximus muscle and is palpable when the
hip is flexed.
The nurse is examining the hip area of a patient and palpates a flat depression on the
upper, lateral side of the thigh when the patient is standing. The nurse interprets this
finding as the:
The greater trochanter of the femur is palpated when the person is standing, and it
appears as a flat depression on the upper lateral side of the thigh.
2. The iliac crest is the upper part of the hip bone;
3. the ischial tuberosity lies under the gluteus maximus muscle and is
palpable when the hip is flexed; and the gluteus muscle is part of the buttocks.
The ankle joint is the articulation of the tibia, fibula, and:
The ankle or tibiotalar joint is the articulation of the tibia, fibula, and talus. The nurse
is explaining the mechanism of the growth of long bones to a mother of a toddler.
Where does lengthening of the bones occur?
Lengthening occurs at the epiphyses, or growth plates. The other options are not
correct.
A woman who is 8 months pregnant comments that she has noticed a change in her
posture and is having lower back pain. The nurse tells her that during pregnancy,
women have a posture shift to compensate for the enlarging fetus. This shift in
posture is known as:
Lordosis compensates for the enlarging fetus, which would shift the center of balance
forward. This shift in balance, in turn, creates a strain on the low back muscles, felt as
low back pain during late pregnancy by some women.
2. Scoliosis is lateral curvature of portions of the spine;
3. ankylosis is extreme flexion of the wrist, as observed with severe
rheumatoid arthritis; and
4. kyphosis is an enhanced thoracic curvature of the spine.
An 85-year-old patient comments during his annual physical examination that he
seems to be getting shorter as he ages. The nurse should explain that decreased
height occurs with aging because:
1. Postural changes are evident with aging; decreased height is most
noticeable and is due to shortening of the vertebral column.
2. Long bones do not shorten with age.
3. Intervertebral disks actually get thinner with age.
4. Subcutaneous fat is not lost but is redistributed to the abdomen and hips. A
patient has been diagnosed with osteoporosis and asks the nurse, "What is
osteoporosis?" The nurse explains that osteoporosis is defined as:
After age 40 years, a loss of bone matrix (resorption) occurs more rapidly than new
bone formation. The net effect is a gradual loss of bone density, or osteoporosis.
The nurse is teaching a class on preventing osteoporosis to a group of perimenopausal
women. Which of these actions is the best way to prevent or delay bone loss in this
group?
Physical activity, such as fast walking, delays or prevents bone loss in perimenopausal
women. The faster the pace of walking, the higher the preventive effect is on the risk of
hip fracture.
A teenage girl has arrived complaining of pain in her left wrist. She was playing
basketball when she fell and landed on her left hand. The nurse examines her hand
and would expect a fracture if the girl complains of a: A fracture causes sharp pain
that increases with movement.
A patient is complaining of pain in his joints that is worse in the morning, better after he
moves around for a while, and then gets worse again if he sits for long periods. The
nurse should assess for other signs of what problem?
Rheumatoid arthritis is worse in the morning when a person arises. Movement
increases most joint pain, except the pain with rheumatoid arthritis, which
decreases with movement.
A patient states, "I can hear a crunching or grating sound when I kneel." She also
states that "it is very difficult to get out of bed in the morning because of stiffness
and pain in my joints." The nurse should assess for signs of what problem?
Crepitation is an audible and palpable crunching or grating that accompanies movement
and occurs when articular surfaces in the joints are roughened, as with rheumatoid
arthritis. The other options are not correct.
A patient is able to flex his right arm forward without difficulty or pain but is unable to
abduct his arm because of pain and muscle spasms. The nurse should suspect:
Rotator cuff lesions may limit range of motion and cause pain and muscle spasms
during abduction, whereas forward flexion remains fairly normal. A professional
tennis player comes into the clinic complaining of a sore elbow. The nurse will
assess for tenderness at the:
The epicondyles, the head of the radius, and the tendons are common sites of
inflammation and local tenderness, commonly referred to as tennis elbow. The other
locations are not affected.
The nurse suspects that a patient has carpal tunnel syndrome and wants to perform the
Phalen test. To perform this test, the nurse should instruct the patient to:
For the Phalen test, the nurse should ask the person to hold both hands back to back
while flexing the wrists 90 degrees. Acute flexion of the wrist for 60 seconds produces
no symptoms in the normal hand. The Phalen test reproduces numbness and burning in
a person with carpal tunnel syndrome. An 80-year-old woman is visiting the clinic for
a checkup. She states, "I can't walk as much as I used to." The nurse is observing for
motor dysfunction in her hip and should ask her to:
abduct her hip while she is lying on her back. Limited abduction of the hip while
supine is the most common motion dysfunction found in hip disease. The nurse has
completed the musculoskeletal examination of a patient's knee and has found a
positive bulge sign. The nurse interprets this finding to indicate:
A positive bulge sign confirms the presence of swelling caused by fluid in the
suprapatellar pouch.
During an examination, the nurse asks a patient to bend forward from the waist and
notices that the patient has lateral tilting. When his leg is raised straight up, the patient
complains of a pain going down his buttock into his leg. The nurse suspects:
Lateral tilting and sciatic pain with straight leg raising are findings that occur with a
herniated nucleus pulposus.
The nurse is examining a 3-month-old infant. While the nurse holds his or her thumbs on
the infant's inner mid thighs and the fingers on the outside of the infant's hips, touching
the greater trochanter, the nurse adducts the legs
until the his or her thumbs touch and then abducts the legs until the infant's knees
touch the table. The nurse does not notice any "clunking" sounds and is confident to
record a:
1. negative ortolans sign. Normally, this maneuver feels smooth and has no
sound.
2. With a positive Ortolani sign, however, the nurse will feel and hear a "clunk," as
the head of the femur pops back into place. A positive Ortolani sign also reflects hip
instability.
3. The Allis test also tests for hip dislocation but is performed by comparing leg
lengths.
During a neonatal examination, the nurse notices that the newborn infant has six
toes. This finding is documented as:
1. Polydactyly is the presence of extra fingers or toes.
2. Syndactyly is webbing between adjacent fingers or toes.
A mother brings her newborn baby boy in for a checkup; she tells the nurse that he
does not seem to be moving his right arm as much as his left and that he seems to
have pain when she lifts him up under the arms. The nurse suspects a fractured
clavicle and would observe for:
For a fractured clavicle, the nurse should observe for limited arm range of motion and
unilateral response to the Moro reflex.
A 40-year-old man has come into the clinic with complaints of extreme pain in his
toes. The nurse notices that his toes are slightly swollen, reddened, and warm to the
touch. His complaints would suggest:
acute gout consist of redness, swelling, heat, and extreme pain like a continuous
throbbing. Gout is a metabolic disorder of disturbed purine metabolism, associated with
elevated serum uric acid.
A young swimmer comes to the sports clinic complaining of a very sore shoulder. He
was running at the pool, slipped on some wet concrete, and tried to catch himself with
his outstretched hand. He landed on his outstretched hand and has not been able to
move his shoulder since. The nurse suspects:
A dislocated shoulder occurs with trauma involving abduction, extension, and external
rotation
A 68-year-old woman has come in for an assessment of her rheumatoid arthritis, and
the nurse notices raised, firm, nontender nodules at the olecranon bursa and along the
ulna. These nodules are most commonly diagnosed as:
Subcutaneous nodules- these are raised, firm, and nontender and occur with rheumatoid
arthritis in the olecranon bursa and along the extensor surface of the ulna
A woman who has had rheumatoid arthritis for years is starting to notice that her fingers
are drifting to the side. The nurse knows that this condition is commonly referred to as:
Ulnar deviation . Fingers drift to the ulnar side because of stretching of the articular
capsule and muscle imbalance caused by chronic rheumatoid arthritis. A radial drift is
not observed.
A patient who has had rheumatoid arthritis for years comes to the clinic to ask about
changes in her fingers. The nurse will assess for signs of what problems?
Changes in the fingers caused by chronic rheumatoid arthritis include swan- neck and
boutonniere deformities.
2. Heberden nodes and Bouchard nodules are associated with osteoarthritis.
3. Dupuytren contractures of the digits occur because of chronic hyperplasia of the
palmar fascia
A patient's annual physical examination reveals a lateral curvature of the thoracic
and lumbar segments of his spine; however, this curvature disappears with forward
bending. The nurse knows that this abnormality of the spine is called:
1. Functional scoliosis - it is flexible and apparent with standing but
disappears with forward bending.
2. Structural scoliosis is fixed; the curvature shows both when standing and when
bending forward.
A 14-year-old boy who has been diagnosed with Osgood-Schlatter disease reports painful
swelling just below the knee for the past 5 months. Which response by the nurse is
appropriate?
"Your disease is due to repeated stress on the patellar tendon. It is usually self-limited,
and your symptoms should resolve with rest." Osgood-Schlatter disease is a painful
swelling of the tibial tubercle just below the knee and most likely due to repeated stress
on the patellar tendon. It is usually self- limited, occurring during rapid growth and
most often in boys. The symptoms resolve with rest. The other responses are not
appropriate.
When assessing muscle strength, the nurse observes that a patient has complete range
of motion against gravity with full resistance. What grade of muscle strength should
the nurse record using a 0- to 5-point scale?
Complete range of motion against gravity is normal muscle strength and is recorded as
grade 5 muscle strength.
The nurse is examining a 6-month-old infant and places the infant's feet flat on the
table and flexes his knees up. The nurse notes that the right knee is significantly lower
than the left. Which of these statements is true of this finding?
Finding one knee significantly lower than the other is a positive Allis sign and suggests
hip dislocation. Normally, the tops of the knees are at the same elevation.
The nurse is assessing a 1-week-old infant and is testing his muscle strength. The nurse
lifts the infant with hands under the axillae and notices that the infant starts to "slip"
between the hands. The nurse should asses that
An infant who starts to "slip" between the nurse's hands shows weakness of the
shoulder muscles. An infant with normal muscle strength wedges securely between
the nurse's hands.
nurse is examining a 2-month-old infant and notices asymmetry of the
infant's gluteal folds. The nurse should assess for other signs of what
disorder?
hip dislocation-Unequal gluteal folds may accompany hip dislocation after 2 to 3
months of age, but some asymmetry may occur in healthy children.
Further assessment is needed. The other responses are not correct.
The nurse should use which test to check for large amounts of fluid around the patella?
Ballottement of the patella is reliable when large amounts of fluid are present.
2. The Tinel sign and the Phalen test are used to check for carpal tunnel
syndrome.
3. The McMurray test is used to test the knee for a torn meniscus.
A patient tells the nurse that, "All my life I've been called 'knock knees'." The nurse
knows that another term for knock knees is:
Genu valgum is also known as knock knees and is present when more than
2.5 cm is between the medial malleoli when the knees are together.
A man who has had gout for several years comes to the clinic with a problem with his
toe. On examination, the nurse notices the presence of hard, painless nodules over the
great toe; one has burst open with a chalky discharge. This finding is known as:
1. Tophi are collections of monosodium urate crystals resulting from chronic gout in and
around the joint that cause extreme swelling and joint deformity. They appear as hard,
painless nodules (tophi) over the metatarsophalangeal joint of the first toe and they
sometimes burst with a chalky discharge
When performing a musculoskeletal assessment, the nurse knows that the correct
approach for the examination should be
The musculoskeletal assessment should be performed in an orderly approach,
head to toe, proximal to distal, from the midline outward.
...
...
The nurse is assessing the joints of a woman who has stated, "I have a
long family history of arthritis, and my joints hurt." The nurse
suspects that she has osteoarthritis. name different symptoms
asymmetric joint involvement, pain with motion of affected joints, affected
joints are swollen with hard bony protuberances
you have just assessed the muscle strength of your patient. you
determine that the patient's knee strength is fair and exhibits active
motion with gravity. you should document the patient's muscle
strength as:
3/5
you are assessing your patient for range of motion movement. you ask him
to move his arm in towards the center of his body. this movement is called:
adduction
a female patient is 8 months pregnant. she comments that she has noticed a
change in posture and is having lower back pain. the nurse tells her that
during pregnancy women have a posture shift to
compensate for the enlarging fetus. this shift is posture is known as:
lordosis
the nurse is checking the range of motion in a patient's knee and knows that
the knee is capable of which movements?
flexion and extension
the nurse notes that there is an audible clicking sound when the patient
opens and closes the mouth. what is the appropriate response of the nurse
at this time?
documenting this finding as expected if no other signs or symptoms are
found
the nurse asks the patient to rest the left arm on a table and to move
the lower arm so that the palm of the hand is up and then down. what
motion is the nurse testing?
supination and pronation of the elbow
upon inspection of a patient's hands, the nurse notices ulnar deviation and
swan-neck deformities bilaterally. the nurses correlates this finding with
which disorder?
rheumatoid arthritis
when performing a musculoskeletal assessment, the nurse knows the
correct approach for the examination should be:
proximal to distal
a patient is able to flex his right arm forward without difficulty or pain;
but is unable to abduct his arm because of pain and muscle spasms. the
nurse should suspect:
rotator cuff injury
the nurse is assessing the joints of a woman who has stated, "I have a
long family history of arthritis, and my joints hurt." the nurse suspects
that she has osteoarthritis. which of these is a symptom of osteoarthritis?
Terms in this set (47)
Original

two parts of the nervous system are the:


ANS: central and peripheral.
The cns includes the brain and spinal cord. The pns 12 pairs of cranial nerves, 31 pairs of
spinal nerves, and branches.
The wife of a 65-year-old man has change in personality and ability to understand. He also
cries. The nurse recalls that the cerebral lobe responsible for these behaviors is the lobe.
ANS: frontal

The frontal lobe has areas concerned with personality, behavior, emotions, and intellectual
function. The parietal lobe has areas concerned with sensation; the
occipital lobe is responsible for visual reception; and the temporal lobe is concerned with
hearing, taste and smell.
Which of these statements concerning areas of the brain is true?
ANS: The hypothalamus controls temperature emotions and regulates sleep.

The cerebellum-motor coordination, equilibrium, balance. The basal


ganglia control autonomic movements of the body.
The area of the nervous system that is responsible for reflexes ANS: spinal
cord.
a nurse accidentally received a prick from an improperly capped needle. To interpret this
sensation?
ANS: Lateral spinothalamic tract, thalamus, and sensory cortex

.
A patient with lack of oxygen, yet pain in his chest and possibly the shoulder, arms,
ANS: The sensory cortex does not have the ability to localize pain in the heart, so the pain is
felt elsewhere.
The ability that humans have to perform very skilled movements such as writing is
controlled by the:
ANS: corticospinal tract.

Corticospinal fibers mediate voluntary movement, particularly very skilled, discrete, purposeful
movements, such as writing.
A 30-year-old woman tells the nurse that she has been very unsteady and has had difficulty in
maintaining her balance.
ANS: Cerebellum
coordinates movement, maintains equilibrium, and helps maintain posture. Which of
these statements about the peripheral nervous system is correct?
ANS: The peripheral nerves carry input to the central nervous system by afferent fibers and
away by efferent fibers.
A patient has a severed spinal nerve as a result of trauma.
ANS: The adjacent spinal nerves will continue to carry sensations for the dermatome.

A dermatome is a circumscribed skin area that is supplied mainly from one spinal cord
segment through a particular spinal nerve.
A 21-year-old patient has a head injury resulting from trauma. what is expected when testing
the patient's deep tendon reflexes?
ANS: Reflexes will be normal.
A reflex is a defense mechanism of the nervous system.
A mother of a 1-month-old infant asks the nurse why it takes so long for infants to learn to roll
over.
ANS: myelin is needed to conduct the impulses, and the neurons of a newborn are not yet
myelinated.
During an assessment of an 80-year-old patient, the nurse notices slightly impaired tactile
sensation. All other neurologic findings are normal.
ANS: normal changes due to aging.
A 70 yr woman tells the nurse that every time she gets up in the morning. she gets "really dizzy"
ANS: "You need to get up slowly when you've been lying or sitting."
During the history, a patient tells the nurse that "it feels like the room is spinning around me."
The nurse would document this as:
ANS: vertigo.
Dizziness is a lightheaded,
Syncope is a sudden loss of consciousness. .
When taking the history on a patient with a seizure disorder, the nurse assesses whether the
patient has an aura.
ANS: "Do you have any warning sign before your seizure starts?"

Aura is a subjective sensation that precedes a seizure; it could be auditory, visual, or motor.
While obtaining a history of a 3-month-old infant, the nurse asks about the infant's ability to
suck and grasp mother's finger.
ANS: Reflexes
In 74-year-old patient the nurse notes that he drinks alcohol daily and that he has noticed a tremor
in his hands.
ANS: "Does the tremor change when you drink the alcohol?"

Senile tremor is relieved by alcohol, although this is not a recommended treatment.


the nurse finds the following: asymmetry when the patient smiles uneven lifting of eyebrows,
sagging of the lower eyelids, and escape of air when the nurse presses against the right puffed
cheek.
ANS: Motor component of VII
The nurse is testing the function of cranial nerve XI. intact?
ANS: moves the head and shoulders against resistance with equal strength.

Checking the patient's ability to hear CN VIII. stick out


the tongue checks the function of CN XII. Testing the
eyes CN III, IV, and VI.
assessment of a "healthy" 35-year-old patient, the The nurse then moves each extremity through
full range of motion.
ANS: Mild, even resistance to movement
When the nurse asks a 68-year-old patient to stand with feet together and his eyes closed, he
starts to sway and moves his feet farther apart.
ANS: positive Romberg sign.
Positive Romberg sign is loss of balance that is increased by closing of the eyes. Ataxia is
unsteady gait.
Homans' sign is used to test the legs for deep vein thrombosis.
a 29-year-old woman who visits the clinic complaining of "always dropping things and
falling down." the woman is unable to pat both her knees.
ANS: Dysfunction of the cerebellum
During the history of a 78-year-old man, he occasionally has short-term memory loss and
confusion: "He can't even remember how to button his shirt."
ANS: Before testing, the nurse would assess the patient's mental status and ability to follow
directions at this time.
The assessment of a 60-year-old patient has taken longer than anticipated. In testing his pain
perception the nurse decides to complete the test as quickly as possible. When the nurse
applies the sharp point of the pin on his arm several times, he is only able to identify these as
one "very sharp prick." What would be the most accurate explanation for this?
ANS: This is most likely the result of the summation effect.

Let at least 2 seconds elapse between each stimulus to avoid summation. With summation,
frequent consecutive stimuli are perceived as one strong stimulus. The other responses are
incorrect.
The nurse is performing a neurologic assessment on a 41-year-old woman with a history of
diabetes. When testing her ability to feel the vibrations of a tuning fork, the nurse notices that
the patient is unable to feel vibrations on the great toe or ankle bilaterally, but she is able to
feel vibrations on both patellae. Given this information, what would the nurse suspect?
ANS: Peripheral neuropathy
Loss of vibration sense occurs with peripheral neuropathy (e.g., diabetes and alcoholism).
Peripheral neuropathy is worse at the feet and gradually improves as the examiner moves up
the leg, as opposed to a specific nerve lesion, which has a clear zone of deficit for its
dermatome.
The nurse places a key in the hand of a patient and he identifies it as a penny. What term would
the nurse use to describe this finding?
ANS: Astereognosis

Stereognosis is the person's ability to recognize objects by feeling their forms, sizes, and
weights. Astereognosis is an inability to identify objects correctly, and it occurs in sensory
cortex lesions. .
In the assessment of a 1-month-old infant, the nurse notices a lack of response to noise or
stimulation. The mother reports that in the last week he has been sleeping all the time, and
when he is awake all he does is cry. The nurse hears that the infant's cries are very high
pitched and shrill. What should be the nurse's appropriate response to these findings?
ANS: Refer the infant for further testing.

A high-pitched, shrill cry or cat-sounding screech occurs with central nervous system damage.
Lethargy, hyporeactivity, hyperirritability, and parent's report of significant change in behavior
all warrant referral. The other options are not correct responses.
Which of these tests would the nurse use to check the motor coordination of an 11- month-old
infant?
Denver II- to screen gross and fine motor coordination, the nurse should use the Denver II
While assessing a 7-month-old infant, the nurse makes a loud noise and notices the following
response: abduction and flexion of arms and legs; fanning of fingers, and curling of index and
thumb in a C position followed by infant bringing in arms and legs to body. What does the
nurse know about this response?
ANS: This reflex should have disappeared between 1 and 4 months of age.

The Moro reflex is present at birth and disappears at 1 to 4 months.


To test for gross motor skill and coordination of a 6-year-old child, which of these techniques
would be appropriate?
ANS: Ask child to hop on one foot.
During the assessment of an 80-year-old patient, the nurse notices that his hands show tremors
when he reaches for something and his head is always nodding. There is no associated rigidity
with movement. Which of these statements is most accurate?
ANS: These are normal findings resulting from aging.

Senile tremors occasionally occur. These benign tremors include an intention tremor of the
hands, head nodding (as if saying yes or no), and tongue protrusion. Tremors associated with
Parkinson disease include rigidity, slowness, and weakness of voluntary movement. The other
responses are incorrect.
While the nurse is taking the history of a 68-year-old patient who sustained a head injury 3
days earlier, he tells the nurse that he is on a cruise ship and is 30 years old. The nurse knows
that this finding is indicative of:
ANS: decreased level of consciousness.

A change in consciousness may be subtle. The nurse should notice any decreasing level of
consciousness, disorientation, memory loss, uncooperative behavior, or even complacency in
a previously combative person. The other responses are incorrect.
The nurse is caring for a patient who has just had neurosurgery. To assess for increased
intracranial pressure, what would the nurse include in the assessment?
ANS: Level of consciousness, motor function, pupillary response, and vital signs

.
assessment of a 22-year who has a head injury, the nurse notices the right pupil is fully dilated
and nonreactive, left pupil is 4 mm and reacts to light.
ANS: Increased intracranial pressure

.
During an assessment of a 62-year-old man the nurse notices the patient has a stooped
posture, shuffling walk with short steps, flat facial expression, and pill- rolling finger
movements.
ANS: parkinsonism.
32-year-old patient with head injury, the patient responds to pain by extending, adducting, and
irotating his arms. palms pronate and his lower extremities extend with plantar flexion.
ANS: This is a very ominous sign and may indicate brainstem injury.
In a person with an upper motor neuron lesion such as a cerebrovascular accident, which of these
physical assessment findings should the nurse expect to see?
ANS: Hyperreflexia

Hyperreflexia, diminished or absent superficial reflexes, and increased muscle tone or


spasticity can be expected with upper motor neuron lesions.
A 59-year-old patient has a herniated intervertebral disk. ANS:
Hyporeflexia

With a herniated intervertebral disk or lower motor neuron lesion there is loss of tone,
flaccidity, atrophy,
A patient is not able to perform rapid alternating movements such as patting her knees rapidly.
The nurse should document this as:
ANS: the presence of dysdiadochokinesia.

Slow clumsy movements and the inability to perform rapid alternating movements are
condition of dysdiadochokinesia.
Ataxia is uncoordinated or unsteady gait.
Astereognosis is the inability to identify an object by feeling it.
Kinesthesia is the person's ability to perceive passive movement of the extremities, or the loss
of position sense.
determining whether a person is oriented to his or her surroundings will test ? ANS:
Cerebrum

The cerebral cortex is responsible for thought, memory, reasoning, sensation, and voluntary
movement.
During an examination, the nurse notices severe nystagmus in both eyes of a patient.
Which of these conclusions by the nurse is correct?
This may indicate disease of the cerebellum or brainstem.

End-point nystagmus at an extreme lateral gaze occurs normally. The


nurse knows that testing kinesthesia is a test of a person's: ANS: position
sense.

Kinesthesia, or position sense, is passive movements of the extremities.


Using the Glasgow Coma Scale, which number indicates that the patient is in a coma?
ANS: 6

A fully alert, normal person has a score of 15, whereas a score of 7 or less reflects coma on
the Glasgow Coma Scale.
a patient who has a late-stage brain tumor. the patient's toes fan out, and the big toe shows
dorsiflexion. The nurse interprets this result as:
ANS: a positive Babinski's sign, which is abnormal for adults.
Dorsiflexion of the big toe and fanning of all toes is a positive Babinski's sign, also called
"upgoing toes
A 69-year-old patient has been admitted to an adult psychiatric unit because his wife thinks
he is getting more and more confused. Alzheimer's disease?
ANS: Difficulty performing familiar tasks, such as placing a telephone call Misplacing
items,
Rapid mood swings, from calm to tears,
Getting lost in one's own neighborhood
During the assessment of deep tendon reflexes, the nurse finds that a patient's responses are
normal bilaterally. What number is used to indicate "normal" deep tendon reflexes when
the documenting this finding. +
ANS: 2

Responses to assessment of deep tendon reflexes are graded on a 4-point scale. A rating of 2+
indicates normal or average response. A rating of 0 indicates no response, and a rating of 4+
indicates very brisk, hyperactive response with clonus, which is indicative of disease.
$2.99
STUDY GUIDE
Neuro System55 Terms
starliightz
4 Chapter 23 JARVIS 256 Terms

Quizlet links
https://quizlet.com/44318852/male-genitalia-hernias-bates-chapter-13-flash-cards/
https://quizlet.com/144854947/6020-final-review-flash-cards/

https://quizlet.com/144855534/health-assessment-final-exam-review-jarvis-6th-ed-flash-cards/

https://quizlet.com/281097103/bates-chapter-14-female-genitalia-flash-cards/

https://quizlet.com/169764569/ch-12-skin-hair-and-nails-flash-cards/
https://quizlet.com/69387169/sexually-transmitted-diseases-flash-cards/
https://quizlet.com/72074567/assessment-test2-skin-etc-flash-cards/
https://quizlet.com/144248445/musculoskeletal-and-nervous-system-flash-cards/

During an examination the nurse observes a female patient's vestibule and


expects to see the
Cervix
urethral meatus and vaginal orifice
During a speculum inspection of the vagina, the nurse would expect to see what at
the end of the vaginal canal?
The uterus is usually positioned tilting forward and superior to the bladder.
This position is Anteverted and anteflexed

A postmenopausal woman should be aware that she is at increased risk for dyspareunia
because of decreased vaginal secretions.
A 54 year old woman who has just completed menopause is in the clinic today for a yearly
physical examination. Which of these statements should the nurse include in patient education?
Stress incontinence
A 52 year old patient states that when she sneezes or coughs she "wets herself a little." She is
very concerned that something may be wrong with her. The nurse suspects that the problem is
The nurse is preparing to interview a postmenopausal woman. Which of these
statements is true with regard to the history of a postmenopausal woman?
The nurse should ask a postmenopausal woman if she ever has vaginal bleeding
Multiple nontender sebaceous cysts.
The nurse has just completed an inspection of a nulliparous woman's external genitalia. Which
of these would be a description of a finding within normal limits?
The nurse is preparing for an internal genitalia examination of a woman. Which
order of the examination is correct?
Speculum, bimanual, rectovaginal
Candidiasis
During a vaginal examination of a 38 year old woman, the nurse notices that the vulva and
vagina are erythematous and edematous with thick, white, curdlike
discharge adhering to the vaginal walls. The woman reports intense pruritus and thick white
discharge from her vagina. The nurse knows that these history and physical examination
findings are most consistent with which of these conditions?
Pelvic inflammatory disease
A 25 year old woman comes to the emergency department with a sudden fever of 101 degrees
F and abdominal pain. Upon examination, the nurse notices that she has a rigid, boardlike
lower abdominal musculature. When the nurse tries to perform a vaginal examination, the
patient has severe pain when the uterus and cervix are moved. The nurse knows that these
signs and symptoms are suggestive of -
FACT #1: The lymphatics of the testes drain into the abdominal lymph nodes
FACT #2: The cremaster muscle contracts in response to cold and draws the testicles
closer to the body.
A male patient with possible fertitility problems asks the nurse where sperm is
produced. The nurse knows that sperm production occurs in the –
Testes
A 62 year old man states that his doctor told him that he has an "inguinal
hernia." He asks the nurse to explain what a hernia is. The nurse should
explain that a hernia is a loop of bowel protruding through a weak spot in the
abdominal muscles.
When the nurse is performing a genital examination on a male patient, the patient
has an erection. The nurse's most appropriate action or response is to- reassure
the patient that
This is a normal response and then continue with the examination
Smegma may be present under the foreskin of an uncircumcised male.
The nurse is examining the glans and knows that which of these is a normal finding for this
area?
Hypospadias
When performing a genitourinary assessment, the nurse notices that the urethral meatus is
positioned ventrally. This finding is
Men with a history of cryptorchidism are at greatest risk for development of testicular cancer.
The nurse is aware that which of these statements is true regarding the incidence of testicular
cancer?
The nurse is describing how to perform a testicular self-examination to a patient.
Which of these statements is most appropriate?
If you notice an enlarged testicle or a painless lump, call you health care
provider.

When performing a genital assessment on a middle-aged man, the nurse


notices multiple soft, moist, painless papules in the shape of cauliflower-
like patches scattered across the shaft of the penis. These lesions are
characteristic of
Genital wart
pulling"
A 15 yearin old
theboy
scrotal area.
is seen in On
theexamination the nurseofpalpates
clinic for complaints a soft,
"dull pain and

irregular mass posterior to and above the testis on the left. The mass
collapses when the patient is supine and refills when he is upright. This
description is
Variocele

ADDITIONAL QUESTIONS

YOU ARE PERFORMING A PAP, HOW CAN YOU MAKE THE PERSON MORE
COMFORTABLE
- HAVE THEM PUT ARMS BEHIND HEAD
- RAISE HEAD AND SHOULDER TO MAINTAIN EYE CONTACT

How venous blood returns back to the heart

- I put it uses positive pressure

WHAT IS ASSOCIATED WITH


RA
- Swan neck
What does a majenta tongue mean

- I put iron deficiency anemia… not really sure

A long term homeless person what is the cause of his problems

- Mentioned lack of vitamin d and calcium


- Lack of calcim
- Lack of iron

There is no pulse felt in tibial pulse at 0/0-4 what should be pulse be

- 2+ is what I put

A guy is lifting something heavy and has incontinence


- Stress incontinence is what I put

A patient has pain while lying in bed, relieved by dangling legs what does he have

- Venous or arterial ? (I put venous I believe)

What is a characteristic of venous insufficiency

- I put brown pigmentation

Pt has low albumin what can you conclude

- I put that there are other causes for low albumin

A patient with chronic emphysema and bronchitis what would you expect to

- see It had clubbing, scleroedema 2 others

Pt has a nevi that has changed, what is most concerning about the findings

- Color variation is what I put


- A 65-year-old patient remarks that she just can’t believe that her
breasts sag so much. She states it must be from lack of exercise.
What explanation should the nurse offer her?
After menopause, the glandular and fat tissue atrophies, causing breast size and
elasticity to diminish, resulting in breasts that sag.

- The mother of a 10-year-old boy asks the nurse to discuss


the recognition of puberty. The nurse should reply by
saying:
“Puberty usually begins about age fifteen.”
“The first sign of puberty is enlargement of the testes.”
“Penis size does not increase until about the age of sixteen.”
"The development of pubic hair precedes testicular or penis enlargement.”
4. During an examination, the nurse notes severe nystagmus in both eyes of
a patient. Which of the following conclusions is correct?
1. This is a normal occurrence.
2. This may indicate disease of the cerebellum or brainstem.
3. This is a sign that the patient is nervous about the examination.
4. This indicates a visual problem and a referral to an ophthalmologist is indicated.
End-point nystagmus at an extreme lateral gaze occurs normally. Assess any
other nystagmus carefully. Severe nystagmus occurs with disease of the vestibular
system, cerebellum, or brainstem.
1. A 43-year-old woman is at the clinic for a routine examination. She
reports that she has had a breast lump in her right breast for years.
Recently, it has begun to change in consistency and is becoming
harder. She reports that 5 years ago her physician evaluated the lump
and determined that it “was nothing to worry about.” The examination
validates the presence of a mass in the right upper outer quadrant at 1
o’clock, approximately 5 cm from the nipple. It is firm, mobile,
nontender, with borders that are not well defined. The nurse’s
recommendation to her is:
“Because of the change in consistency of the lump, it should be further evaluated
by a physician.”
1. A patient's mother has noticed that her son, who has been to a new
babysitter, has some blisters and scabs on his face and buttocks. On
examination, the nurse notices moist, thin-roofed vesicles with a
thin erythematous base and suspects:
Impetigo
9. A 16-yr-old girl is being seen at clinic for gastrointestinal
complaints+weight loss. Nurse determines that many of her complaints
may be related to erratic eating patterns, eating predominantly fast
foods,
+ high caffeine intake. In this situation, which is most appropriate when
collecting current dietary intake information?
I picked keep a log of 2 work days and 1 weekend

10. A patient has a positive Homans' sign. The nurse knows that a
positive Homans' sign may indicate:
deep vein thrombosis. (this wasn’t an option …. It said venous insufficiency or
thrombophlebitis )

9. A 15-year-old boy is seen in the clinic for complaints of “dull pain and
pulling” in the scrotal area. On examination the nurse palpates a soft,
irregular mass posterior to and above the testis on the left. This mass
collapses when the patient is supine and refills when he is upright. This
description is consistent with:
varicocele
10.The nurse is bathing an 80-year-old man and notices that his skin is
wrinkled, thin, lax, and dry. This finding would be related to which
factor?
An increased loss of elastin and a decrease in subcutaneous fat in the elderly
17. In assessment of 1-month-old, nurse notes a lack of response to noise or
stimulation. mother reports that in the last week he has been sleeping all the
time + when awake all he does is cry. nurse hears that infant’s cries are very
high pitched and shrill. What would be nurse’s appropriate response?
1. Refer the infant for further testing.
2. Talk with the mother about eating habits.
3. Nothing; these are expected findings for an infant this age.
4. Tell the mother to bring the baby back in a week for a recheck.
A high-pitched shrill cry or cat-sounding screech occurs with central nervous
system damage. Lethargy, hyporeactivity, hyperirritability, and parent’s report of
significant change in behavior all warrant referral.
11.A 14-year-old girl is anxious about not having reached menarche. When
taking the history, the nurse should ascertain which of the following?
The age:
she began to develop breasts
12. During an examination, the nurse notes a supernumerary nipple just under
the patient’s left breast. The patient tells the nurse that she always thought
it was a mole. Which statement about this finding is correct?
It is a normal variation and not a significant finding
23. In a person with an upper motor neuron lesion such as a cerebrovascular
accident, which of the following physical assessment findings would the nurse
expect to see?
1.Hyperreflexia
2.Fasciculations
3.Loss of muscle tone and flaccidity
4.Atrophy and wasting of the muscles
Hyperreflexia, diminished or absent superficial reflexes, increased muscle tone or
spasticity can be expected with upper motor neuron lesions.
27. uring an examination, you note that a male patient has a red, round,
superficial ulcer with a yellowish-serous discharge on his penis. Upon
palpation, you note a nontender base that feels like a small button between
your thumb and fingers. At this point you suspect that this patient has:
syphilitic chancre

28. During an interview, a patient reveals that she is pregnant. She states that
she is not sure whether she will breastfeed her baby and asks for some
information about this. Which of these statements by the nurse is accurate
with regard to breastfeeding?
“Breastfeeding provides the perfect food and antibodies for your baby.”
or Breast-feeding may reduce the risk of breast cancer.
30. The nurse practitioner is examining only the rectal area of a woman and
should place the woman in what position?
The nurse should place the female patient in lithotomy position if examining
genitalia as well; use the left lateral decubitus
33. A man found wandering in park at 2AM has been brought to emergency
department for examination because he said he fell+hit head. During
examination, nurse asks him to use his index finger to touch nurse’s finger,
then own nose, then nurse’s finger again (moved to a different location).
patient is clumsy, unable to follow the instructions, + overshoots mark,
missing finger. nurse suspects...
1.Cerebral injury
2.Cerebrovascular accident
3.Acute alcohol intoxication
4.Peripheral neuropathy
. During an assessment, the nurse notes that a patient's left arm is swollen
from the shoulder down to the fingers, with nonpitting edema. The right arm
is normal. The patient had a mastectomy 1 year ago. The nurse suspects
which problem?
Lymphedema
. The assessment of an 80-year-old patient, the nurse notes that his hands show
tremors when he reaches for something and that his head is always nodding.
There is no associated rigidity with movement. Which of the following
statements is most accurate?
these findings are normal

44. Which of the following statements is true regarding the arterial system?
The arterial system is a high-pressure system.

45. A patient who is visiting the clinic complains of having "stomach pains
for 2 weeks" and describes his stools as being "soft and black" for about the
last 10 days. He denies taking any medications. The NP is aware that these
symptoms are most indicative of:
occult blood resulting from gastrointestinal bleeding.

47. When assessing a patient the nurse practitioner documents the left femoral
pulse as 0/0-4+. Which of the following findings would the nurse practitioner
expect at the dorsalis pedis pulse?
0/0-4+ Pulsations are graded on a four-point scale: 0, absent; 1+, weak; 2+,
normal; 3+, increased; 4+, bounding. If a pulse is absent at the femoral site, one
would expect the dorsalis pedis pulse to be absent also.
49. The nurse practitioner is doing an assessment on a 29-year-old woman
who visits the clinic complaining of "always dropping things and falling
down." While testing rapid alternating movements, the nurse practitioner
notices that the woman is unable to pat both her knees. Her response is
very slow and she misses frequently. What might the nurse practitioner
suspect?
Dysfunction of the cerebellum
In rapid, alternating movements, slow, clumsy, and sloppy response occurs with
cerebellar disease.

50. The wife of a 65 year old man tells the nurse that she is concerned
because she has noted a change in her husband's personality and ability to
understand. He also cries and becomes angry very easily. The nurse recalls
that the cerebral lobe responsible for these behaviors is which of the
following?
Frontal
51. A male patient with possible fertility problems asks the nurse where
sperm is produced. The nurse knows that sperm production occurs in the:
testes.

52. Which of the following statements reflects the best approach to teaching
a woman about breast self-examination (BSE)?
"BSE on a monthly basis will help you feel familiar with your own breasts and
their normal variations."

53. When assessing a patient's pulse, the nurse practitioner notes that the
amplitude is weaker during inspiration and stronger during expiration.
When the nurse practitioner measures the blood pressure, the reading
decreases 20 mm Hg during inspiration and increases with expiration. This
patient is experiencing:
pulsus paradoxus.
57. Assessment of a 60-yr-old patient has taken longer than anticipated. in
testing pain perception nurse decides to complete the test as quickly as
possible. When nurse applies sharp point of pin on his arm several times, he is
only able to identify these as one 'very sharp prick.' most accurate
explanation?

1. has hyperesthesia as a result of aging process


2. most likely the result of the summation effect.
3. The nurse was probably not poking hard enough with the pin in the other areas.
4. The patient most likely has analgesia in some areas of arm and hyperalgesia in
others.
ANS: 2 Let at least 2 seconds elapse between each stimulus to avoid summation.
With summation, frequent consecutive stimuli are perceived as one strong
stimulus.

57. Assessing a 7-month-old infant you make a loud noise and note the
following response: Abduction and flexion of arms and legs; fanning of
fingers and curling of index finger and thumb in C-position; followed by
infant bringing in arms and legs to body. What do you know about this?
1. This could indicate brachial nerve palsy
2. This is an expected startle response at this age
3. This reflex should disappear between 1 and 4 months of age
4. It is normal as long as movements are symmetrical bilaterally
61. Which of the following veins are responsible for most of the venous
return in the arm?
1. Deep veins
2. Ulnar veins
3.Subclavian veins
4.Superficial veins

62. A man who has had gout for several years comes to the clinic with a
"problem with my toe." On examination, the nurse practitioner notes the
presence of hard, painless nodules over the great toe; one had burst open with
a chalky discharge. This finding is known as:
Tophi are collections of sodium urate crystals resulting from chronic gout in and
around the joint that cause extreme swelling and joint deformity. They appear as
hard, painless nodules (tophi) over the metatarsophalangeal joint of the first toe
and they sometimes burst with a chalky discharge

64. A 70-year-old patient is scheduled for open-heart surgery. The surgeon


plans to use the great saphenous vein for the coronary bypass grafts. The
patient asks, “What happens to my circulation when the veins are
removed?” The nurse should reply:
“Because the deeper veins in your leg are in good condition, this vein can be
removed without harming your circulation.”

65. When observing the vestibule, the nurse practitioner should be able to
see the:
Urethral meatus and vaginal orifice
66. When the nurse is conducting sexual history from a male adolescent,
which statement would be most appropriate to use at the beginning of
the interview?
Often boys your age have questions about sexual activity
70. Mrs. A has had arthritis for years and is starting to notice that her fingers
are drifting to this side. This is commonly referred to as:
ulnar deviation
72. A 14-year-old boy who has been diagnosed with Osgood-Schlatter disease
reports painful swelling just below the knee for the past 5 months. Which
response by the nurse practitioner is appropriate?
"Your disease is due to repeated stress on the patellar tendon. It is usually self-
limited, and your symptoms should resolve with rest."

75. The nurse practitioner has completed the musculoskeletal examination


of a patient’s knee and has found a positive bulge sign. The nurse suspects:
Swelling from fluid in the suprapatellar pouch

76. Which of the following statements is true with regard to the history of a
postmenopausal woman?
ASK IF SHE EVER HAS VAGINAL BLEEDING
78. During a physical examination, a 45-year-old woman states that she has
had a crusty, itchy rash on her breast for about 2 weeks. In trying to find the
cause of the rash, which of these would be important for the nurse to
determine?
Where did it first appear—on the nipple, the areola, or the surrounding skin?
??? CONCERN FOR PAGETS DISEASE
80. The patient is in her first trimester of pregnancy. She complains of feeling
nauseated and has vomited on occasion. She tells the nurse that she did not
have this with her first pregnancy. She asks the nurse, "What is causing this
and when will it end?" How should the nurse respond?

The nausea is caused by elevated levels of progesterone and estrogen, and the
nausea should end once her body adjusts to the increased hormone levels.
81. A professional tennis player comes in complaining of a sore elbow.
You suspect that he has tenderness at:
the medial and lateral epicondyle

85. In examining a 70-year-old male patient, the nurse notices that he


has bilateral gynecomastia. Which of the following describes the nurse’s
best course of action?
Explain that this condition may be the result of hormonal changes and recommend
that he see his physician.
88. During an annual check-up of a 55 y/o patient, the nurse discusses the
early detection measures for colon cancer. The nurse should mention the
need for a
colonoscopy every 10 years

89. What are the two main parts of the nervous system?
Central Nervous System (CNS) and Peripheral Nervous System (PNS)

90. While obtaining a history of a 3-month old infant from the mother,
the nurse practitioner asks about the baby's ability to suck and grasp the
mother's finger. What is the nurse practitioner assessing?
REFLEXES

You might also like